Test for A/H and Psych 4

Réussis tes devoirs et examens dès maintenant avec Quizwiz!

Acemprosate (Campral)

Given to people recovering from alcohol abuse/dependence. Reduces the physical and emotional discomfort during the first weeks/months of sobriety. Discomfort: sweating, anxiety, and sleep disturbances. PERSONS WITH RENAL FAILURE CANNOT TAKE THIS DRUG. Side effects: diarrhea, nausea, flatulence, and pruritus.

Magnetic resonance imaging (MRI)

Imaging method: Radio waves from the brain detected from magnet Results: Structural image Duration: 45 minutes Person must lay in a small, closed chamber and remain motionless during the procedure. People with claustrophobia or increased anxiety may have to be sedated before the procedure

Positron Emission Tomography (PET) and Single Photon Emission Computed Tomography (SPECT)

Imaging method: Radioactive tracer injected into bloodstream and monitored as client performs activities Results: Functional; primarily used for research Duration: 2-3 hours (PET), 1-2 hours (SPECT) PET scan yields a better resolution with sharper and clearer pictures.

Computed tomography (CT/CAT)

Imaging method: Serial x-ray of brain Results: Structural image Duration: 20-40 minutes Person must lay motionless on a "stretcher" that passes through tunnel-like ring.

Limbic system

Includes the amygdala, hippocampus, hypothalamus, and thalamus. Disturbances have been linked to memory-loss accompanied with dementia and poor impulse and emotional control with psychotic and manic behaviour.

CNS (central nervous system)

Includes the brain, spinal cord, and associated nerves which control voluntary act.

Other side effects of antipsychotics

Increased prolactin blood levels, breast enlargement/tenderness, increased risk for breast cancer, diminished libido, erectile dysfunction, menstruation irregularities, orgasmic difficulties, and weight gain. More prone to metabolic syndrome. Atypical antipsychotics: greater risk for weight gain and hyperglycemia and thus diabetes.

Brain stem includes

Mid-brain, pons, medulla oblongata, and cranial nerves III-XII

Antisocial personality disorders are assessed with which tool?

Milton Clinical Multiaxial Inventory (MCMI)

Bupropion (Wellbutrin)

Risk for seizures when dose >450mg/day, cranial trauma, Hx of seizures, withdrawal from alcohol, addiction to opiates, cocaine, or stimulants/anorectics

Preferred drugs for clients with high suicide risk

SSRIs are the choice as they are not lethal with overdose but are only effective for mild to moderate depression.

An older adult patient sought care for the treatment of a swollen, painful knee joint. Diagnostic imaging and culturing of synovial fluid resulted in a diagnosis of septic arthritis. The nurse should prioritize which of the following aspects of care? A) Administration of oral and IV corticosteroids as ordered B) Prevention of falls and pathologic fractures C) Maintenance of adequate serum levels of vitamin D D) Intravenous administration of antibiotics

D

An orthopedic nurse is caring for a patient who is postoperative day one following foot surgery. What nursing intervention should be included in the patients subsequent care? A) Dressing changes should not be performed unless there are clear signs of infection. B) The surgical site can be soaked in warm bath water for up to 5 minutes. C) The surgical site should be cleansed with hydrogen peroxide once daily. D) The foot should be elevated in order to prevent edema.

D

Serotonin syndrome

Results from taking SSRIs and MOAIs together.

Extrapyramidal Symptoms (EPS)

Reversible movement disorders induced by antipsychotic or neuroleptic medication (i.e. acute dystonia, pseudoparkinsonism, & akathisia)

Neuroleptic Malignant Syndrome

Rigidity, high fever, autonomic instability (unstable BP , diaphoresis, pallor, delirium), and elevated level of enzymes, particularly creatine phosphokinase. Patients are usually confused and mute, may fluctuate from agitation to stupor. Dehydration, poor nutrition, and concurrent medical illness may increase risk for NMS. Treatment includes discontinuing antipsychotic immediately and supportive medical care.

Amygdala

Two Lima bean-sized neural clusters in the limbic system; linked to emotional arousal and memory.

Cerebrum

Two hemispheres. Hemispheres are divided into four lobes: Frontal, parietal, temporal, and occipital.

Glucosamine/Chondroitin

arthritis supplement

Asking the client to complete serial sevens assesses what?

concentration

dopaminergics

convert to dopamine

neoglisms

inverted words that have meaning for that client only

waxy flexibility

maintenance of posture or position over time even when it is awkward or uncomfortable

NSAIDs

medication that exerts analgesic and anti-inflammatory actions

Anticholinergic uses

motion sickness

Dystonia

muscular rigidity and cramping, a stiff or thick tongue with difficulty swallowing, and, in severe cases, laryngospasm and respiratory difficulty

dopaminergic system

neuronal systems that use dopamine as their major neurotransmitter

NSAID

non-steroidal anti-inflammatory drug

Thicket for

parkinson's with difficulty swallowing

Computed tomography (CT or CAT scan)

procedure in which a precise x-ray beam takes cross sectional images (slices) layer by layer

Muscle Relaxants

relieve muscle spasms and stiffness, MS, spinal cord injury, cerebral palsy, stroke

automatisms

repeated, seemingly purposeless behaviors often indicative of anxiety, such as drumming fingers, twisting locks of hair, or tapping the foot; unconscious mannerism

blunted

showing little or slow to respond

Flat

showing no facial expression

anticoagulant

slows coagulation and prevents new clots from forming

thought blocking

stopping abruptly in the middle of a sentence or train of thought; sometimes client is unable to continue the idea

Immunosuppressants

suppress the immune response to prevent organ rejection after transplantation or slow the progression of autoimmune disease

Rebound

temporary return of symptoms; may be more intense than original symptoms

Potency

the amount of the drug needed to achieve the maximum effect

When considering where to conduct a psychosocial assessment, the nurse can effectively interview which client in the unit's conference room?

the anxious client The nurse should not choose an isolated location such as a conference room for the interview, if the client is unknown to the nurse or has a history of any threatening behavior either to themselves or to others. The anxious client by diagnosis does not present a threat and so is the one best suited for the nurse to use the conference room for the interview.

Akathisia

the intense need to move about. The client appears restless or anxious and agitated, often with a rigid posture or gait and a lack of spontaneous gestures

Efficacy

the maximal therapeutic effect that a drug can achieve

Immunomodulators

treat moderate to severe rheumatoid arthritis by disrupting the inflammatory process and delaying disease progression

Washing of the face often precipitates attacks of

trigeminal neuralgia

Labile

unpredictable and rapid mood swings with no apparent stimuli

antiparkinsonian

used for palliative relief of major symptoms of Parkinson disease (L-Dopa, Requip, Symmetrel, Tasar)

Antivertigo

used to treat dizziness (vertigo)

tangential thinking

wandering off the topic and never providing the information requested

The health care team is caring for a patient with osteomalacia. It has been determined that the osteomalacia is caused by malabsorption. What is the usual treatment for osteomalacia caused by malabsorption? A) Supplemental calcium and increased doses of vitamin D B) Exogenous parathyroid hormone and multivitamins C) Colony-stimulating factors and calcitonin D) Supplemental potassium and pancreatic enzymes

A

21. The nurse is caring for a patient with permanent neurologic impairments resulting from a traumatic head injury. When working with this patient and family, what mutual goal should be prioritized? A) Achieve as high a level of function as possible. B) Enhance the quantity of the patients life. C) Teach the family proper care of the patient. D) Provide community assistance.

A) Achieve as high a level of function as possible.

31. A patient is recovering from intracranial surgery performed approximately 24 hours ago and is complaining of a headache that the patient rates at 8 on a 10-point pain scale. What nursing action is most appropriate? A) Administer morphine sulfate as ordered. B) Reposition the patient in a prone position. C) Apply a hot pack to the patients scalp. D) Implement distraction techniques.

A) Administer morphine sulfate as ordered.

11. A nurse will conduct an influenza vaccination campaign at an extended care facility. The nurse will be administering intramuscular (IM) doses of the vaccine. Of what age-related change should the nurse be aware when planning the appropriate administration of this drug? A) An older patient has less subcutaneous tissue and less muscle mass than a younger patient. B) An older patient has more subcutaneous tissue and less durable skin than a younger patient. C) An older patient has more superficial and tortuous nerve distribution than a younger patient. D) An older patient has a higher risk of bleeding after an IM injection than a younger patient.

A) An older patient has less subcutaneous tissue and less muscle mass than a younger patient.

A patient diagnosed with Bell's palsy is being cared for on an outpatient basis. During health education, the nurse should promote which of the following actions? A) Applying a protective eye shield at night B) Chewing on the affected side to prevent unilateral neglect C) Avoiding the use of analgesics whenever possible D) Avoiding brushing the teeth

A) Applying a protective eye shield at night

Midbrain

Connects connect the pons and cerebellum with the cerebrum. It includes the reticular activating system and the extra-pyramidal system.

Neurobiological causes of mental illness

Genetics and heredity Stress and the immune system (psychoimmunology) Infection

A nurse is performing a psychosocial assessment of the client. Which questions asked by the nurse can be identified as open ended? Select all that apply.

"How can we help you?" "How did your problems begin?"

Which client statement best demonstrates a healthy relationship with family?

"I feel better after I visit with my Mom." The relationship with others is important to mental health. Feeling better after having contact with a particular person demonstrates a healthy relationship

The nurse has entered a hospital client's room and asked the client if the client plans to attend the morning's scheduled group life-skills session. Which response should signal the presence of thought blocking to the nurse?

"I might. I'll give it some..."

Foods containing Tyramine

*Mature/aged cheeses (all cheese except cottage cheese, cream cheese, ricotta cheese, & processed cheese slices) *Aged meats (such as pepperoni, salami, mortadella, summer sausage, beef logs, meat extracts, & similar products) *Italian broad beans (lava), bean curd (tofu), banana peel, overripe fruit, & avacado *All tap beers & microbrewery beer *Sauerkraut, soy sauce or soybean condiments, or marmite *Yogurt, sour cream, peanuts, Brewer's yeast, and MSG

During assessment of a client with schizophrenia, the nurse notes the client has ideas of reference. Which statement of the client would have led the nurse to conclude this?

"The news of the terrorist attack is directed to me. The terrorists are trying to warn me."

The nurse is performing a psychosocial assessment of a client. Which questions should the nurse ask to assess the client's self-concept? Select all that apply.

"What do you do when you have a problem?" "How do you think you look today?"

When assessing orientation, the nurse completes the assessment by asking which questions? Select all that apply.

"What is your name?" "What day of the week is it?" "Can you tell me where you are?"

Antidepressant groups

1. Tricyclic and related cyclic 2. Selective serotonin re-uptake inhibitors (SSRIs) 3. MAO inhibitors 4. Others: Desvenlafaxine (Pristiq)

Ginkgo Biloba uses

1. dementia 2. short term memory loss 3. vertigo 4.peripheral vascular diseases 5. depression 6. sexual dysfunction

A 32-year-old patient comes to the clinic complaining of shoulder tenderness, pain, and limited movement. Upon assessment the nurse finds edema. An MRI shows hemorrhage of the rotator cuff tendons and the patient is diagnosed with impingement syndrome. What action should the nurse recommend in order to promote healing? A) Support the affected arm on pillows at night. B) Take prescribed corticosteroids as ordered. C) Put the shoulder through its full range of motion 3 times daily. D) Keep the affected arm in a sling for 2 to 4 weeks.

A

A nurse is assessing a patient who reports a throbbing, burning sensation in the right foot. The patient states that the pain is worst during the day but notes that the pain is relieved with rest. The nurse should recognize the signs and symptoms of what health problem? A) Mortons neuroma B) Pescavus C) Hallux valgus D) Onychocryptosis

A

A nurse is caring for a patient who is 12 hours postoperative following foot surgery. The nurse assesses the presence of edema in the foot. What nursing measure will the nurse implement to control the edema? A) Elevate the foot on several pillows. B) Apply warm compresses intermittently to the surgical area. C) Administer a loop diuretic as ordered. D) Increase circulation through frequent ambulation.

A

A nurse is caring for an adult patient diagnosed with a back strain. What health education should the nurse provide to this patient? A) Avoid lifting more than one-third of body weight without assistance. B) Focus on using back muscles efficiently when lifting heavy objects. C) Lift objects while holding the object a safe distance from the body. D) Tighten the abdominal muscles and lock the knees when lifting of an object.

A

A nurse is collaborating with the physical therapist to plan the care of a patient with osteomyelitis. What principle should guide the management of activity and mobility in this patient? A) Stress on the weakened bone must be avoided. B) Increased heart rate enhances perfusion and bone healing. C) Bed rest results in improved outcomes in patients with osteomyelitis. D) Maintenance of baseline ADLs is the primary goal during osteomyelitis treatment.

A

A nurse is planning the care of an older adult patient with osteomalacia. What action should the nurse recommend in order to promote vitamin D synthesis? A) Ensuring adequate exposure to sunlight B) Eating a low-purine diet C) Performing cardiovascular exercise while avoiding weight-bearing exercises D) Taking thyroid supplements as ordered

A

A nurse is providing discharge teaching for a patient who underwent foot surgery. The nurse is collaborating with the occupational therapist and discussing the use of assistive devices. On what variables does the choice of assistive devices primarily depend? A) Patients general condition, balance, and weight-bearing prescription B) Patients general condition, strength, and gender C) Patients motivation, age, and weight-bearing prescription D) Patients occupation, motivation, and age

A

A nurse is teaching a patient with osteomalacia about the role of diet. What would be the best choice for breakfast for a patient with osteomalacia? A) Cereal with milk, a scrambled egg, and grapefruit B) Poached eggs with sausage and toast C) Waffles with fresh strawberries and powdered sugar D) A bagel topped with butter and jam with a side dish of grapes

A

A patient presents to a clinic complaining of a leg ulcer that isnt healing; subsequent diagnostic testing suggests osteomyelitis. The nurse is aware that the most common pathogen to cause osteomyelitis is what? A) Staphylococcus aureus B) Proteus C) Pseudomonas D) Escherichia coli

A

A patient tells the nurse that he has pain and numbness to his thumb, first finger, and second finger of the right hand. The nurse discovers that the patient is employed as an auto mechanic, and that the pain is increased while working. This may indicate that the patient could possibly have what health problem? A) Carpel tunnel syndrome B) Tendonitis C) Impingement syndrome D) Dupuytrens contracture

A

A patients electronic health record notes that the patient has hallux valgus. What signs and symptoms would the nurse expect this patient to manifest? A) Deviation of a great toe laterally B) Abnormal flexion of the great toe C) An exaggerated arch of the foot D) Fusion of the toe joints

A

An 80-year-old man in a long-term care facility has a chronic leg ulcer and states that the area has become increasingly painful in recent days. The nurse notes that the site is now swollen and warm to the touch. The patient should undergo diagnostic testing for what health problem? A) Osteomyelitis B) Osteoporosis C) Osteomalacia D) Septic arthritis

A

An elderly female with osteoporosis has been hospitalized. Prior to discharge, when teaching the patient, the nurse should include information about which major complication of osteoporosis? A) Bone fracture B) Loss of estrogen C) Negative calcium balance D) Dowagers hump

A

An older adult womans current medication regimen includes alendronate (Fosamax). What outcome would indicate successful therapy? A) Increased bone mass B) Resolution of infection C) Relief of bone pain D) Absence of tumor spread

A

4. The nurse is caring for a 65-year-old patient who has previously been diagnosed with hypertension. Which of the following blood pressure readings represents the threshold between high-normal blood pressure and hypertension? A) 140/90 mm Hg B) 145/95 mm Hg C) 150/100 mm Hg D) 160/100 mm Hg

A) 140/90 mm Hg

Antidepressants

A class of psychotropic medications used for the treatment of depression, anxiety disorders, depressive phases of bipolar, and psychotic depression. Off label use for: chronic pain, migraines, peripheral and diabetic neuropathies, sleep apnea, dermatological disorders, panic disorders, and eating disorders.

Metabolic syndrome

A condition that increases the risk for heart disease, diabetes, and stroke. Diagnosed when presenting three or more: 1. Obesity 2. Increased blood pressure 3. High sugar levels 4. High cholesterol (150 mg/dL of triglyceride, less 40mg/dL of HDL for women; 50mg/dL for men)

A nurse is caring for an 86-year-old female patient who has become increasingly frail and unsteady on her feet. During the assessment, the patient indicates that she has fallen three times in the month, though she has not yet suffered an injury. The nurse should take action in the knowledge that this patient is at a high risk for what health problem? A) A hip fracture B) A femoral fracture C) Pelvic dysplasia D) Tearing of a meniscus or bursa

A) A hip fracture

Disulfram (Antabuse)

A deterrent to drinking alcohol, useful for persons motivated to abstain from drinking but are not impulsive. MOST EFFECTIVE IN PEOPLE WITH (NEAR) NORMAL LIVER ENZYME LEVELS. Causes adverse reactions when mixed with alcohol. Symptoms progress fast and lasts 30 - 120 minutes. In the first 5 - 10 minutes you'll see face and body flushing from vasodilation, throbbing headache, sweating, dry mouth, nausea, vomiting, dizziness, and weakness. In severe cases patient may have chest pain, dyspnea, severe hypotension, confusion, or death. Products (i.e. cologne) containing alcohol can cause same effect. Other side effects: halitosis, tremors, impotence, fatigue, and drowsiness. Interferes with: Phenytoin, isoniazide, warfarin, barbiturates, and long acting benzodiazepines.

Single photon emission computed tomography (SPECT)

A diagnostic test used to examine the function of the brain by following the flow of an injected radioactive substance (uses a single photon)

Corticosteroids

A group of hormones, including cortisol, released by the adrenal glands at times of stress

Black box warning

A highlighted box which contains a warning about serious or life-threatening side effects of a drug.

Neuroleptic Malignant Syndrome (NMS)

A potentially fatal, idiosyncratic reaction to an antipsychotic (neuroleptic) drug. Major symptoms are rigidity; high fever; autonomic instability such as unstable BP, diaphoresis, and pallor; delirium; and elevated levels of enzymes (especially creatine phosphokinase). Clients with NMS are usually confused and often mute.

Depot injection

A slow-release, injectable form of antipsychotic medication for maintenance therapy

Magnetic Resonance Imaging (MRI)

A type of body scan where an energy field is created with a huge magnet and radio waves then the energy field is converted to a visual image or scan

The nurse is teaching a patient with Guillain-Barré syndrome about the disease. The patient asks how he can ever recover if demyelination of his nerves is occurring. What would be the nurse's best response? A) "Guillain-Barré spares the Schwann cell, which allows for remyelination in the recovery phase of the disease." B) "In Guillain-Barré, Schwann cells replicate themselves before the disease destroys them, so remyelination is possible." C) "I know you understand that nerve cells do not remyelinate, so the physician is the best one to answer your question." D) "For some reason, in Guillain-Barré, Schwann cells become activated and take over the remyelination process."

A) "Guillain-Barré spares the Schwann cell, which allows for remyelination in the recovery phase of the disease."

40. An adult patient has sought care for the treatment of headaches that have become increasingly severe and frequent over the past several months. Which of the following questions addresses potential etiological factors? Select all that apply? A) Are you exposed to any toxins or chemicals at work? B) How would you describe your ability to cope with stress? C) What medications are you currently taking? D) When was the last time you were hospitalized? E) Does anyone else in your family struggle with headaches?

A) Are you exposed to any toxins or chemicals at work? B) How would you describe your ability to cope with stress? C) What medications are you currently taking? E) Does anyone else in your family struggle with headaches?

23. The nurse is caring for a patient whose recent health history includes an altered LOC. What should be the nurses first action when assessing this patient? A) Assessing the patients verbal response B) Assessing the patients ability to follow complex commands C) Assessing the patients judgment D) Assessing the patients response to pain

A) Assessing the patients verbal response

The nurse caring for a patient in a persistent vegetative state is regularly assessing for potential complications. Complications of neurologic dysfunction for which the nurse should assess include which of the following? Select all that apply. A) Contractures B) Hemorrhage C) Pressure ulcers D) Venous thromboembolism E) Pneumonia

A) Contractures C) Pressure ulcers D) Venous thromboembolism E) Pneumonia

14. The nurse has created a plan of care for a patient who is at risk for increased ICP. The patients care plan should specify monitoring for what early sign of increased ICP? A) Disorientation and restlessness B) Decreased pulse and respirations C) Projectile vomiting D) Loss of corneal reflex

A) Disorientation and restlessness

The nurse is caring for a patient who is hospitalized with an exacerbation of MS. To ensure the patient's safety, what nursing action should be performed? A) Ensure that suction apparatus is set up at the bedside. B) Pad the patient's bed rails. C) Maintain bed rest whenever possible. D) Provide several small meals each day.

A) Ensure that suction apparatus is set up at the bedside.

A patient with MS has been admitted to the hospital following an acute exacerbation. When planning the patient's care, the nurse addresses the need to enhance the patient's bladder control. What aspect of nursing care is most likely to meet this goal? A) Establish a timed voiding schedule. B) Avoid foods that change the pH of urine. C) Perform intermittent catheterization q6h. D) Administer anticholinergic drugs as ordered.

A) Establish a timed voiding schedule.

29. When caring for a patient with increased ICP the nurse knows the importance of monitoring for possible secondary complications, including syndrome of inappropriate antidiuretic hormone (SIADH). What nursing interventions would the nurse most likely initiate if the patient developed SIADH? A) Fluid restriction B) Transfusion of platelets C) Transfusion of fresh frozen plasma (FFP) D) Electrolyte restriction

A) Fluid restriction

5. You are the nurse caring for an 85-year-old patient who has been hospitalized for a fractured radius. The patients daughter has accompanied the patient to the hospital and asks you what her father can do for his very dry skin, which has become susceptible to cracking and shearing. What would be your best response? A) He should likely take showers rather than baths, if possible. B) Make sure that he applies sunscreen each morning. C) Dry skin is an age-related change that is largely inevitable. D) Try to help your father increase his intake of dairy products.

A) He should likely take showers rather than baths, if possible.

2. A gerontologic nurse practitioner provides primary care for a large number of older adults who are living with various forms of cardiovascular disease. This nurse is well aware that heart disease is the leading cause of death in the aged. What is an age-related physiological change that contributes to this trend? A) Heart muscle and arteries lose their elasticity. B) Systolic blood pressure decreases. C) Resting heart rate decreases with age. D) Atrial-septal defects develop with age.

A) Heart muscle and arteries lose their elasticity.

You are the clinic nurse caring for a patient with a recent diagnosis of myasthenia gravis. The patient has begun treatment with pyridostigmine bromide (Mestinon). What change in status would most clearly suggest a therapeutic benefit of this medication? A) Increased muscle strength B) Decreased pain C) Improved GI function D) Improved cognition

A) Increased muscle strength

26. The nurse is caring for a patient who sustained a moderate head injury following a bicycle accident. The nurses most recent assessment reveals that the patients respiratory effort has increased. What is the nurses most appropriate response? A) Inform the care team and assess for further signs of possible increased ICP. B) Administer bronchodilators as ordered and monitor the patients LOC. C) Increase the patients bed height and reassess in 30 minutes. D) Administer a bolus of normal saline as ordered.

A) Inform the care team and assess for further signs of possible increased ICP.

A 35-year-old woman is diagnosed with a peripheral neuropathy. When making her plan of care, the nurse knows to include what in patient teaching? Select all that apply. A) Inspect the lower extremities for skin breakdown. B) Footwear needs to be accurately sized. C) Immediate family members should be screened for the disease. D) Assistive devices may be needed to reduce the risk of falls. E) Dietary modifications are likely necessary.

A) Inspect the lower extremities for skin breakdown. B) Footwear needs to be accurately sized. D) Assistive devices may be needed to reduce the risk of falls.

The nurse is providing patient teaching to a patient with early stage Alzheimers disease (AD) and her family. The patient has been prescribed donepezil hydrochloride (Aricept). What should the nurse explain to the patient and family about this drug? A) It slows the progression of AD. B) It cures AD in a small minority of patients. C) It removes the patients insight that he or she has AD. D) It limits the physical effects of AD and other dementias.

A) It slows the progression of AD.

The nurse is working with a patient who is newly diagnosed with MS. What basic information should the nurse provide to the patient? A) MS is a progressive demyelinating disease of the nervous system. B) MS usually occurs more frequently in men. C) MS typically has an acute onset. D) MS is sometimes caused by a bacterial infection.

A) MS is a progressive demyelinating disease of the nervous system.

12. An elderly patient, while being seen in an urgent care facility for a possible respiratory infection, asks the nurse if Medicare is going to cover the cost of the visit. What information can the nurse give the patient to help allay her concerns? A) Medicare has a copayment for many of the services it covers. This requires the patient to pay a part of the bill. B) Medicare pays for 100% of the cost for acute-care services, so the cost of the visit will be covered. C) Medicare will only pay the cost for acute-care services if the patient has a very low income. D) Medicare will not pay for the cost of acute-care services so the patient will be billed for the services provided.

A) Medicare has a copayment for many of the services it covers. This requires the patient to pay a part of the bill.

38. Falls, which are a major health problem in the elderly population, occur from multifactorial causes. When implementing a comprehensive plan to reduce the incidence of falls on a geriatric unit, what risk factors should nurses identify? Select all that apply. A) Medication effects B) Overdependence on assistive devices C) Poor lighting D) Sensory impairment E) Ineffective use of coping strategies

A) Medication effects C) Poor lighting D) Sensory impairment

37. A hospital patient has experienced a seizure. In the immediate recovery period, what action best protects the patients safety? A) Place the patient in a side-lying position. B) Pad the patients bed rails. C) Administer antianxiety medications as ordered. D) Reassure the patient and family members.

A) Place the patient in a side-lying position.

The nurse is caring for a 77-year-old woman with MS. She states that she is very concerned about the progress of her disease and what the future holds. The nurse should know that elderly patients with MS are known to be particularly concerned about what variables? Select all that apply. A) Possible nursing home placement B) Pain associated with physical therapy C) Increasing disability D) Becoming a burden on the family E) Loss of appetite

A) Possible nursing home placement C) Increasing disability D) Becoming a burden on the family

A patient with diabetes presents to the clinic and is diagnosed with a mononeuropathy. This patient's nursing care should involve which of the following? A) Protection of the affected limb from injury B) Passive and active ROM exercises for the affected limb C) Education about improvements to glycemic control D) Interventions to prevent contractures

A) Protection of the affected limb from injury

33. A patient is postoperative day 1 following intracranial surgery. The nurses assessment reveals that the patients LOC is slightly decreased compared with the day of surgery. What is the nurses best response to this assessment finding? A) Recognize that this may represent the peak of post-surgical cerebral edema. B) Alert the surgeon to the possibility of an intracranial hemorrhage. C) Understand that the surgery may have been unsuccessful. D) Recognize the need to refer the patient to the palliative care team.

A) Recognize that this may represent the peak of post-surgical cerebral edema.

34. Based on a patients vague explanations for recurring injuries, the nurse suspects that a community- dwelling older adult may be the victim of abuse. What is the nurses primary responsibility? A) Report the findings to adult protective services. B) Confront the suspected perpetrator. C) Gather evidence to corroborate the abuse. D) Work with the family to promote healthy conflict resolution.

A) Report the findings to adult protective services.

30. For several years, a community health nurse has been working with a 78-year-old man who requires a wheelchair for mobility. The nurse is aware that the interactions between disabilities and aging are not yet clearly understood. This interaction varies, depending on what variable? A) Socioeconomics B) Ethnicity C) Education D) Pharmacotherapy

A) Socioeconomics

30. The nurse is admitting a patient to the unit who is scheduled for removal of an intracranial mass. What diagnostic procedures might be included in this patients admission orders? Select all that apply. A) Transcranial Doppler flow study B) Cerebral angiography C) MRI D) Cranial radiography E) Electromyelography (EMG)

A) Transcranial Doppler flow study B) Cerebral angiography C) MRI

The nurse is developing a plan of care for a patient with Guillain-Barré syndrome. Which of the following interventions should the nurse prioritize for this patient? A) Using the incentive spirometer as prescribed B) Maintaining the patient on bed rest C) Providing aids to compensate for loss of vision D) Assessing frequently for loss of cognitive function

A) Using the incentive spirometer as prescribed

17. A patient has developed diabetes insipidus after having increased ICP following head trauma. What nursing assessment best addresses this complication? A) Vigilant monitoring of fluid balance B) Continuous BP monitoring C) Serial arterial blood gases (ABGs) D) Monitoring of the patients airway for patency

A) Vigilant monitoring of fluid balance

The nurse is planning discharge education for a patient with trigeminal neuralgia. The nurse knows to include information about factors that precipitate an attack. What would the nurse be correct in teaching the patient to avoid? A) Washing his face B) Exposing his skin to sunlight C) Using artificial tears D) Drinking large amounts of fluids

A) Washing his face

A patient has been admitted to the medical unit for the treatment of Pagets disease. When reviewing the medication administration record, the nurse should anticipate what medications? Select all that apply. A) Calcitonin B) Bisphosphonates C) Alkaline phosphatase D) Calcium gluconate E) Estrogen

A, B

A nurse is providing care for a patient who has a recent diagnosis of Pagets disease. When planning this patients nursing care, interventions should address what nursing diagnoses? Select all that apply. A) Impaired Physical Mobility B) Acute Pain C) Disturbed Auditory D) Sensory Perception Risk for Injury E) Risk for Unstable Blood Glucose

A, B, C, D

A nurse is caring for a patient who is being assessed following complaints of severe and persistent low back pain. The patient is scheduled for diagnostic testing in the morning. Which of the following are appropriate diagnostic tests for assessing low back pain? that apply. A) Computed tomography (CT) B) Angiography C) Magnetic resonance imaging (MRI) D) Ultrasound E) X-ray

A, C, D, E

The nurse is preparing to perform a psychosocial assessment of the client. Which describes the most effective approach taken by the nurse?

Accepting Objective While performing a psychosocial assessment, the nurse should have an accepting and objective or straightforward approach with the client. If the nurse expresses personal opinions to the client or is judgmental, the client may be reluctant to share sensitive information. The nurse should not be emotionally unstable as this would hinder the nurse's ability for proper assessment. If the nurse asks short and crisp questions, the client may think the nurse is hurried and is not interested in listening. This behavior may prevent the client from providing complete information to the nurse.

Non-opioid analgesics

Acetaminophen (Tylenol) Analgesic and antipyretic effects NSAIDs Aspirin & COX-2 inhibitors (Celebrex) Management of pain especially inflammatory conditions (arthritis)

Acute dystonia

Acute muscular rigidity and cramping, a stiff or thick tongue with difficulty swallowing, and in severe cases laryngospasm and respiratory difficulties.

delusion

fixed, false belief that cannot be changed by logical reasoning or evidence

A nurse documents that "the client describes the recent breakup of a dating relationship with an emotionless tone and a flat facial expression." In which section of the mental status exam would the nurse have documented this statement?

Affect Affect refers to a person's emotional expression (in this case, the manner in which the client talks about the client's experiences). Feelings are emotional states or perceptions. Blocking is the interruption of thoughts. Moods are prolonged emotional states expressed by the affect.

Myasthenia gravis is a lower motor neuron disease.

Also need frequent rest periods and myasthenic crisis requires ventilation assistance

26. The nurse is planning the care of a patient who has been recently diagnosed with a cerebellar tumor. Due to the location of this patient's tumor, the nurse should implement measures to prevent what complication? A) Falls B) Audio hallucinations C) Respiratory depression D) Labile BP

Ans: A Feedback: A cerebellar tumor causes dizziness, an ataxic or staggering gait with a tendency to fall toward the side of the lesion, and marked muscle incoordination. Because of this, the patient faces a high risk of falls. Hallucinations and unstable vital signs are not closely associated with cerebellar tumors.

A nurse is assessing a patient with an acoustic neuroma who has been recently admitted to an oncology unit. What symptoms is the nurse likely to find during the initial assessment? A) Loss of hearing, tinnitus, and vertigo B) Loss of vision, change in mental status, and hyperthermia C) Loss of hearing, increased sodium retention, and hypertension D) Loss of vision, headache, and tachycardia

Ans: A Feedback: An acoustic neuroma is a tumor of the eighth cranial nerve, the cranial nerve most responsible for hearing and balance. The patient with an acoustic neuroma usually experiences loss of hearing, tinnitus, and episodes of vertigo and staggering gait. Acoustic neuromas do not cause loss of vision, increased sodium retention, or tachycardia.

19. A patient with Huntington disease has just been admitted to a long-term care facility. The charge nurse is creating a care plan for this patient. Nutritional management for a patient with Huntington disease should be informed by what principle? A) The patient is likely to have an increased appetite. B) The patient is likely to required enzyme supplements. C) The patient will likely require a clear liquid diet. D) The patient will benefit from a low-protein diet.

Ans: A Feedback: Due to the continuous involuntary movements, patients will have a ravenous appetite. Despite this ravenous appetite, patients usually become emaciated and exhausted. As the disease progresses, patients experience difficulty in swallowing and thin liquids should be avoided. Protein will not be limited with this disease. Enzyme supplements are not normally required.

23. A gerontologic nurse is advocating for diagnostic testing of an 81-year-old patient who is experiencing personality changes. The nurse is aware of what factor that is known to affect the diagnosis and treatment of brain tumors in older adults? A) The effects of brain tumors are often attributed to the cognitive effects of aging. B) Brain tumors in older adults do not normally produce focal effects. C) Older adults typically have numerous benign brain tumors by the eighth decade of life. D) Brain tumors cannot normally be treated in patient over age 75.

Ans: A Feedback: In older adult patients, early signs and symptoms of intracranial tumors can be easily overlooked or incorrectly attributed to cognitive and neurologic changes associated with normal aging. Brain tumors are not normally benign and they produce focal effects in all patients. Treatment options are not dependent primarily on age.

25. A male patient presents at the free clinic with complaints of impotency. Upon physical examination, the nurse practitioner notes the presence of hypogonadism. What diagnosis should the nurse suspect? A) Prolactinoma B) Angioma C) Glioma D) Adrenocorticotropic hormone (ACTH)-producing adenoma

Ans: A Feedback: Male patients with prolactinomas may present with impotence and hypogonadism. An ACTH-producing adenoma would cause acromegaly. The scenario contains insufficient information to know if the tumor is an angioma, glioma, or neuroma.

30. An older adult has encouraged her husband to visit their primary care provider, stating that she is concerned that he may have Parkinson's disease. Which of the wife's descriptions of her husband's health and function is most suggestive of Parkinson's disease? A) "Lately he seems to move far more slowly than he ever has in the past." B) "He often complains that his joints are terribly stiff when he wakes up in the morning." C) "He's forgotten the names of some people that we've known for years." D) "He's losing weight even though he has a ravenous appetite."

Ans: A Feedback: Parkinson's disease is characterized by bradykinesia. It does not manifest as memory loss, increased appetite, or joint stiffness.

37. A patient with a new diagnosis of amyotrophic lateral sclerosis (ALS) is overwhelmed by his diagnosis and the known complications of the disease. How can the patient best make known his wishes for care as his disease progresses? A) Prepare an advance directive. B) Designate a most responsible physician (MRP) early in the course of the disease. C) Collaborate with representatives from the Amyotrophic Lateral Sclerosis Association. D) Ensure that witnesses are present when he provides instruction.

Ans: A Feedback: Patients with ALS are encouraged to complete an advance directive or "living will" to preserve their autonomy in decision making. None of the other listed actions constitutes a legally binding statement of end-of-life care.

27. A patient has been admitted to the neurologic ICU with a diagnosis of a brain tumor. The patient is scheduled to have a tumor resection/removal in the morning. Which of the following assessment parameters should the nurse include in the initial assessment? A) Gag reflex B) Deep tendon reflexes C) Abdominal girth D) Hearing acuity

Ans: A Feedback: Preoperatively, the gag reflex and ability to swallow are evaluated. In patients with diminished gag response, care includes teaching the patient to direct food and fluids toward the unaffected side, having the patient sit upright to eat, offering a semisoft diet, and having suction readily available. Deep tendon reflexes, abdominal girth, and hearing acuity are less commonly affected by brain tumors and do not affect the risk for aspiration.

40. A nurse is planning discharge education for a patient who underwent a cervical diskectomy. What strategies would the nurse assess that would aid in planning discharge teaching? A) Care of the cervical collar B) Technique for performing neck ROM exercises C) Home assessment of ABGs D) Techniques for restoring nerve function

Ans: A Feedback: Prior to discharge, the nurse should assess the patient's use and care of the cervical collar. Neck ROM exercises would be contraindicated and ABGs cannot be assessed in the home. Nerve function is not compromised by a diskectomy.

33. The nurse caring for a patient diagnosed with Parkinson's disease has prepared a plan of care that would include what goal? A) Promoting effective communication B) Controlling diarrhea C) Preventing cognitive decline D) Managing choreiform movements

Ans: A Feedback: The goals for the patient may include improving functional mobility, maintaining independence in ADLs, achieving adequate bowel elimination, attaining and maintaining acceptable nutritional status, achieving effective communication, and developing positive coping mechanisms. Constipation is more likely than diarrhea and cognition largely remains intact. Choreiform movements are related to Huntington disease.

A patient with suspected Parkinson's disease is initially being assessed by the nurse. When is the best time to assess for the presence of a tremor? A) When the patient is resting B) When the patient is ambulating C) When the patient is preparing his or her meal tray to eat D) When the patient is participating in occupational therapy

Ans: A Feedback: The tremor is present while the patient is at rest; it increases when the patient is walking, concentrating, or feeling anxious. Resting tremor characteristically disappears with purposeful movement, but is evident when the extremities are motionless. Consequently, the nurse should assess for the presence of a tremor when the patient is not performing deliberate actions.

36. A family member of a patient diagnosed with Huntington disease calls you at the clinic. She is requesting help from the Huntington's Disease Society of America. What kind of help can this patient and family receive from this organization? Select all that apply. A) Information about this disease B) Referrals C) Public education D) Individual assessments E) Appraisals of research studies

Ans: A, B, C Feedback: The Huntington's Disease Society of America helps patients and families by providing information, referrals, family and public education, and support for research. It does not provide individual assessments or appraisals of individual research studies.

29. A patient with an inoperable brain tumor has been told that he has a short life expectancy. On what aspects of assessment and care should the home health nurse focus? Select all that apply. A) Pain control B) Management of treatment complications C) Interpretation of diagnostic tests D) Assistance with self-care E) Administration of treatments

Ans: A, B, D, E Feedback: Home care needs and interventions focus on four major areas: palliation of symptoms and pain control, assistance in self-care, control of treatment complications, and administration of specific forms of treatment, such as parenteral nutrition. Interpretation of diagnostic tests is normally beyond the purview of the nurse.

34. The nurse is caring for a patient diagnosed with Parkinson's disease. The patient is having increasing problems with rising from the sitting to the standing position. What should the nurse suggest to the patient to use that will aid in getting from the sitting to the standing position as well as aid in improving bowel elimination? A) Use of a bedpan B) Use of a raised toilet seat C) Sitting quietly on the toilet every 2 hours D) Following the outlined bowel program

Ans: B Feedback: A raised toilet seat is useful, because the patient has difficulty in moving from a standing to a sitting position. A handicapped toilet is not high enough and will not aid in improving bowel elimination. Sitting quietly on the toilet every 2 hours will not aid in getting from the sitting to standing position; neither will following the outlined bowel program.

18. A patient has just returned to the unit from the PACU after surgery for a tumor within the spine. The patient complains of pain. When positioning the patient for comfort and to reduce injury to the surgical site, the nurse will position to patient in what position? A) In the high Fowler's position B) In a flat side-lying position C) In the Trendelenberg position D) In the reverse Trendelenberg position

Ans: B Feedback: After spinal surgery, the bed is usually kept flat initially. The side-lying position is usually the most comfortable because this position imposes the least pressure on the surgical site. The Fowler's position, Trendelenberg position, and reverse Trendelenberg position are inappropriate for this patient because they would result in increased pain and complications.

38. The nurse is caring for a patient who is scheduled for a cervical discectomy the following day. During health education, the patient should be made aware of what potential complications? A) Vertebral fracture B) Hematoma at the surgical site C) Scoliosis D) Renal trauma

Ans: B Feedback: Based on all the assessment data, the potential complications of diskectomy may include hematoma at the surgical site, resulting in cord compression and neurologic deficit and recurrent or persistent pain after surgery. Renal trauma and fractures are unlikely; scoliosis is a congenital malformation of the spine.

14. A patient diagnosed with a pituitary adenoma has arrived on the neurologic unit. When planning the patient's care, the nurse should be aware that the effects of the tumor will primarily depend on what variable? A) Whether the tumor utilizes aerobic or anaerobic respiration B) The specific hormones secreted by the tumor C) The patient's pre-existing health status D) Whether the tumor is primary or the result of metastasis

Ans: B Feedback: Functioning pituitary tumors can produce one or more hormones normally produced by the anterior pituitary and the effects of the tumor depend largely on the identity of these hormones. This variable is more significant than the patient's health status or whether the tumor is primary versus secondary. Anaerobic and aerobic respiration is not relevant.

16. The nurse in an extended care facility is planning the daily activities of a patient with postpolio syndrome. The nurse recognizes the patient will best benefit from physical therapy when it is scheduled at what time? A) Immediately after meals B) In the morning C) Before bedtime D) In the early evening

Ans: B Feedback: Important activities for patients with postpolio syndrome should be planned for the morning, as fatigue often increases in the afternoon and evening.

A 25-year-old female patient with brain metastases is considering her life expectancy after her most recent meeting with her oncologist. Based on the fact that the patient is not receiving treatment for her brain metastases, what is the nurse's most appropriate action? A) Promoting the patient's functional status and ADLs B) Ensuring that the patient receives adequate palliative care C) Ensuring that the family does not tell the patient that her condition is terminal D) Promoting adherence to the prescribed medication regimen

Ans: B Feedback: Patients with intracerebral metastases who are not treated have a steady downhill course with a limited survival time, whereas those who are treated may survive for slightly longer periods, but for most cure is not possible. Palliative care is thus necessary. This is a priority over promotion of function and the family should not normally withhold information from the patient. Adherence to medications such as analgesics is important, but palliative care is a high priority.

13. A patient has been admitted to the neurologic unit for the treatment of a newly diagnosed brain tumor. The patient has just exhibited seizure activity for the first time. What is the nurse's priority response to this event? A) Identify the triggers that precipitated the seizure. B) Implement precautions to ensure the patient's safety. C) Teach the patient's family about the relationship between brain tumors and seizure activity. D) Ensure that the patient is housed in a private room.

Ans: B Feedback: Patients with seizures are carefully monitored and protected from injury. Patient safety is a priority over health education, even though this is appropriate and necessary. Specific triggers may or may not be evident; identifying these is not the highest priority. A private room is preferable, but not absolutely necessary.

6. The nurse is caring for a boy who has muscular dystrophy. When planning assistance with the patient's ADLs, what goal should the nurse prioritize? A) Promoting the patient's recovery from the disease B) Maximizing the patient's level of function C) Ensuring the patient's adherence to treatment D) Fostering the family's participation in care

Ans: B Feedback: Priority for the care of the child with muscular dystrophy is the need to maximize the patient's level of function. Family participation is also important, but should be guided by this goal. Adherence is not a central goal, even though it is highly beneficial, and the disease is not curable.

17. A patient newly diagnosed with a cervical disk herniation is receiving health education from the clinic nurse. What conservative management measures should the nurse teach the patient to implement? A) Perform active ROM exercises three times daily. B) Sleep on a firm mattress. C) Apply cool compresses to the back of the neck daily. D) Wear the cervical collar for at least 2 hours at a time.

Ans: B Feedback: Proper positioning on a firm mattress and bed rest for 1 to 2 days may bring dramatic relief from pain. The patient may need to wear a cervical collar 24 hours a day during the acute phase of pain from a cervical disk herniation. Hot, moist compresses applied to the back of the neck will increase blood flow to the muscles and help relax the spastic muscles.

5. The clinic nurse caring for a patient with Parkinson's disease notes that the patient has been taking levodopa and carbidopa (Sinemet) for 7 years. For what common side effect of Sinemet would the nurse assesses this patient? A) Pruritus B) Dyskinesia C) Lactose intolerance D) Diarrhea

Ans: B Feedback: Within 5 to 10 years of taking levodopa, most patients develop a response to the medication characterized by dyskinesia (abnormal involuntary movements). Another potential complication of long-term dopaminergic medication use is neuroleptic malignant syndrome characterized by severe rigidity, stupor, and hyperthermia. Side effects of long-term Sinemet therapy are not pruritus, lactose intolerance, or diarrhea.

8. A patient with Parkinson's disease is undergoing a swallowing assessment because she has recently developed adventitious lung sounds. The patient's nutritional needs should be met by what method? A) Total parenteral nutrition (TPN) B) Provision of a low-residue diet C) Semisolid food with thick liquids D) Minced foods and a fluid restriction

Ans: C Feedback: A semisolid diet with thick liquids is easier for a patient with swallowing difficulties to consume than is a solid diet. Low-residue foods and fluid restriction are unnecessary and counterproductive to the patient's nutritional status. The patient's status does not warrant TPN.

28. A patient with a brain tumor has begun to exhibit signs of cachexia. What subsequent assessment should the nurse prioritize? A) Assessment of peripheral nervous function B) Assessment of cranial nerve function C) Assessment of nutritional status D) Assessment of respiratory status

Ans: C Feedback: Cachexia is a wasting syndrome of weight loss, muscle atrophy, fatigue, weakness, and significant loss of appetite. Consequently, nutritional assessment is paramount.

31. A patient, brought to the clinic by his wife and son, is diagnosed with Huntington disease. When providing anticipatory guidance, the nurse should address the future possibility of what effect of Huntington disease? A) Metastasis B) Risk for stroke C) Emotional and personality changes D) Pathologic bone fractures

Ans: C Feedback: Huntington disease causes profound changes to personality and behavior. It is a nonmalignant disease and stroke is not a central risk. The disease is not associated with pathologic bone fractures.

39. The nurse responds to the call light of a patient who has had a cervical diskectomy earlier in the day. The patient states that she is having severe pain that had a sudden onset. What is the nurse's most appropriate action? A) Palpate the surgical site. B) Remove the dressing to assess the surgical site. C) Call the surgeon to report the patient's pain. D) Administer a dose of an NSAID.

Ans: C Feedback: If the patient experiences a sudden increase in pain, extrusion of the graft may have occurred, requiring reoperation. A sudden increase in pain should be promptly reported to the surgeon. Administration of an NSAID would be an insufficient response and the dressing should not be removed without an order. Palpation could cause further damage.

20. A patient with amyotrophic lateral sclerosis (ALS) is being visited by the home health nurse who is creating a care plan. What nursing diagnosis is most likely for a patient with this condition? A) Chronic confusion B) Impaired urinary elimination C) Impaired verbal communication D) Bowel incontinence

Ans: C Feedback: Impaired communication is an appropriate nursing diagnosis; the voice in patients with ALS assumes a nasal sound and articulation becomes so disrupted that speech is unintelligible. Intellectual function is marginally impaired in patients with late ALS. Usually, the anal and bladder sphincters are intact because the spinal nerves that control muscles of the rectum and urinary bladder are not affected.

35. A patient with Parkinson's disease is experiencing episodes of constipation that are becoming increasingly frequent and severe. The patient states that he has been achieving relief for the past few weeks by using OTC laxatives. How should the nurse respond? A) "It's important to drink plenty of fluids while you're taking laxatives." B) "Make sure that you supplement your laxatives with a nutritious diet." C) "Let's explore other options, because laxatives can have side effects and create dependency." D) "You should ideally be using herbal remedies rather than medications to promote bowel function."

Ans: C Feedback: Laxatives should be avoided in patients with Parkinson's disease due to the risk of adverse effects and dependence. Herbal bowel remedies are not necessarily less risky.

The nurse is writing a care plan for a patient with brain metastases. The nurse decides that an appropriate nursing diagnosis is "anxiety related to lack of control over the health circumstances." In establishing this plan of care for the patient, the nurse should include what intervention? A) The patient will receive antianxiety medications every 4 hours. B) The patient's family will be instructed on planning the patient's care. C) The patient will be encouraged to verbalize concerns related to the disease and its treatment. D) The patient will begin intensive therapy with the goal of distraction.

Ans: C Feedback: Patients need the opportunity to exercise some control over their situation. A sense of mastery can be gained as they learn to understand the disease and its treatment and how to deal with their feelings. Distraction and administering medications will not allow the patient to gain control over anxiety. Delegating planning to the family will not help the patient gain a sense of control and autonomy.

22. The nurse is caring for a patient newly diagnosed with a primary brain tumor. The patient asks the nurse where his tumor came from. What would be the nurse's best response? A) "Your tumor originated from somewhere outside the CNS." B) "Your tumor likely started out in one of your glands." C) "Your tumor originated from cells within your brain itself." D) "Your tumor is from nerve tissue somewhere in your body."

Ans: C Feedback: Primary brain tumors originate from cells and structures within the brain. Secondary brain tumors are metastatic tumors that originate somewhere else in the body. The scenario does not indicate that the patient's tumor is a pituitary tumor or a neuroma.

24. A patient who has been experiencing numerous episodes of unexplained headaches and vomiting has subsequently been referred for testing to rule out a brain tumor. What characteristic of the patient's vomiting is most consistent with a brain tumor? A) The patient's vomiting is accompanied by epistaxis. B) The patient's vomiting does not relieve his nausea. C) The patient's vomiting is unrelated to food intake. D) The patient's emesis is blood-tinged.

Ans: C Feedback: Vomiting is often unrelated to food intake if caused by a brain tumor. The presence or absence of blood is not related to the possible etiology and vomiting may or may not relieve the patient's nausea.

21. The nurse educator is discussing neoplasms with a group of recent graduates. The educator explains that the effects of neoplasms are caused by the compression and infiltration of normal tissue. The physiologic changes that result can cause what pathophysiologic events? Select all that apply. A) Intracranial hemorrhage B) Infection of cerebrospinal fluid C) Increased ICP D) Focal neurologic signs E) Altered pituitary function

Ans: C, D, E Feedback: The effects of neoplasms are caused by the compression and infiltration of tissue. A variety of physiologic changes result, causing any or all of the following pathophysiologic events: increased ICP and cerebral edema, seizure activity and focal neurologic signs, hydrocephalus, and altered pituitary function.

9. While assessing the patient at the beginning of the shift, the nurse inspects a surgical dressing covering the operative site after the patients' cervical diskectomy. The nurse notes that the drainage is 75% saturated with serosanguineous discharge. What is the nurse's most appropriate action? A) Page the physician and report this sign of infection. B) Reinforce the dressing and reassess in 1 to 2 hours. C) Reposition the patient to prevent further hemorrhage. D) Inform the surgeon of the possibility of a dural leak.

Ans: D Feedback: After a cervical diskectomy, the nurse will monitor the operative site and dressing covering this site. Serosanguineous drainage may indicate a dural leak. This constitutes a risk for meningitis, but is not a direct sign of infection. This should be reported to the surgeon, not just reinforced and observed.

11. A patient has just been diagnosed with Parkinson's disease and the nurse is planning the patient's subsequent care for the home setting. What nursing diagnosis should the nurse address when educating the patient's family? A) Risk for infection B) Impaired spontaneous ventilation C) Unilateral neglect D) Risk for injury

Ans: D Feedback: Individuals with Parkinson's disease face a significant risk for injury related to the effects of dyskinesia. Unilateral neglect is not characteristic of the disease, which affects both sides of the body. Parkinson's disease does not directly constitute a risk for infection or impaired respiration.

10. A patient, diagnosed with cancer of the lung, has just been told he has metastases to the brain. What change in health status would the nurse attribute to the patient's metastatic brain disease? A) Chronic pain B) Respiratory distress C) Fixed pupils D) Personality changes

Ans: D Feedback: Neurologic signs and symptoms include headache, gait disturbances, visual impairment, personality changes, altered mentation (memory loss and confusion), focal weakness, paralysis, aphasia, and seizures. Pain, respiratory distress, and fixed pupils are not among the more common neurologic signs and symptoms of metastatic brain disease.

12. The nurse is caring for a patient with Huntington disease who has been admitted to the hospital for treatment of malnutrition. What independent nursing action should be implemented in the patient's plan of care? A) Firmly redirect the patient's head when feeding. B) Administer phenothiazines after each meal as ordered. C) Encourage the patient to keep his or her feeding area clean. D) Apply deep, gentle pressure around the patient's mouth to aid swallowing.

Ans: D Feedback: Nursing interventions for a patient who has inadequate nutritional intake should include the following: Apply deep gentle pressure around the patient's mouth to assist with swallowing, and administer phenothiazines prior to the patient's meal as ordered. The nurse should disregard the mess of the feeding area and treat the person with dignity. Stiffness and turning away by the patient during feeding are uncontrollable choreiform movements and should not be interrupted.

7. A 37-year-old man is brought to the clinic by his wife because he is experiencing loss of motor function and sensation. The physician suspects the patient has a spinal cord tumor and hospitalizes him for diagnostic testing. In light of the need to diagnose spinal cord compression from a tumor, the nurse will most likely prepare the patient for what test? A) Anterior-posterior x-ray B) Ultrasound C) Lumbar puncture D) MRI

Ans: D Feedback: The MRI scan is the most commonly used diagnostic procedure. It is the most sensitive diagnostic tool that is particularly helpful in detecting epidural spinal cord compression and vertebral bone metastases.

32. A patient who was diagnosed with Parkinson's disease several months ago recently began treatment with levodopa-carbidopa. The patient and his family are excited that he has experienced significant symptom relief. The nurse should be aware of what implication of the patient's medication regimen? A) The patient is in a "honeymoon period" when adverse effects of levodopa-carbidopa are not yet evident. B) Benefits of levodopa-carbidopa do not peak until 6 to 9 months after the initiation of treatment. C) The patient's temporary improvement in status is likely unrelated to levodopa-carbidopa. D) Benefits of levodopa-carbidopa often diminish after 1 or 2 years of treatment.

Ans: D Feedback: The beneficial effects of levodopa therapy are most pronounced in the first year or two of treatment. Benefits begin to wane and adverse effects become more severe over time. However, a "honeymoon period" of treatment is not known.

15. A male patient with a metastatic brain tumor is having a generalized seizure and begins vomiting. What should the nurse do first? A) Perform oral suctioning. B) Page the physician. C) Insert a tongue depressor into the patient's mouth. D) Turn the patient on his side.

Ans: D Feedback: The nurse's first response should be to place the patient on his side to prevent him from aspirating emesis. Inserting something into the seizing patient's mouth is no longer part of a seizure protocol. Obtaining supplies to suction the patient would be a delegated task. Paging or calling the physician would only be necessary if this is the patient's first seizure.

A patient who has undergone a lower limb amputation is preparing to be discharged home. What outcome is necessary prior to discharge? A) Patient can demonstrate safe use of assistive devices. B) Patient has a healed, nontender, nonadherent scar. C) Patient can perform activities of daily living independently. D) Patientis free of pain.

Ans: A Feedback: A patient should be able to use assistive devices appropriately and safely prior to discharge. Scar formation will not be complete at the time of hospital discharge. It is anticipated that the patient will require some assistance with ADLs postdischarge. Pain should be well managed, but may or may not be wholly absent.

A patient was fitted with an arm cast after fracturing her humerus. Twelve hours after the application of the cast, the patient tells the nurse that her arm hurts. Analgesics do not relieve the pain. What would be the most appropriate nursing action? A) Prepare the patient for opening or bivalving of the cast. B) Obtain an order for a different analgesic. C) Encourage the patient to wiggle and move the fingers. D) Petal the edges of the patient's cast.

Ans: A Feedback: Acute compartment syndrome involves a sudden and severe decrease in blood flow to the tissues distal to an area of injury that results in ischemic necrosis if prompt, decisive intervention does not occur. Removing or bivalving the cast is necessary to relieve pressure. Ordering different analgesics does not address the underlying problem. Encouraging the patient to move the fingers or perform range-of-motion exercises will not treat or prevent compartment syndrome. Petaling the edges of a cast with tape prevents abrasions and skin breakdown, not compartment syndrome.

A rehabilitation nurse is working with a patient who has had a below-the-knee amputation. The nurse knows the importance of the patient's active participation in self-care. In order to determine the patient's ability to be an active participant in self-care, the nurse should prioritize assessment of what variable? A) The patient's attitude B) The patient's learning style C) The patient's nutritional status D) The patient's presurgical level of function

Ans: A Feedback: Amputation of an extremity affects the patient's ability to provide adequate self-care. The patient is encouraged to be an active participant in self-care. The patient and the nurse need to maintain positive attitudes and to minimize fatigue and frustration during the learning process. Balanced nutrition and the patient's learning style are important variables in the rehabilitation process but the patient's attitude is among the most salient variables. The patient's presurgical level of function may or may not affect participation in rehabilitation.

The surgical nurse is admitting a patient from postanesthetic recovery following the patient's below-the-knee amputation. The nurse recognizes the patient's high risk for postoperative hemorrhage and should keep which of the following at the bedside? A) A tourniquet B) A syringe preloaded with vitamin K C) A unit of packed red blood cells, placed on ice D) A dose of protamine sulfate

Ans: A Feedback: Immediate postoperative bleeding may develop slowly or may take the form of massive hemorrhage resulting from a loosened suture. A large tourniquet should be in plain sight at the patient's bedside so that, if severe bleeding occurs, it can be applied to the residual limb to control the hemorrhage. PRBCs cannot be kept at the bedside. Vitamin K and protamine sulfate are antidotes to warfarin and heparin, but are not administered to treat active postsurgical bleeding.

A 25-year-old man is involved in a motorcycle accident and injures his arm. The physician diagnoses the man with an intra-articular fracture and splints the injury. The nurse implements the teaching plan developed for this patient. What sequela of intra-articular fractures should the nurse describe regarding this patient? A) Post-traumatic arthritis B) Fat embolism syndrome (FES) C) Osteomyelitis D) Compartment syndrome

Ans: A Feedback: Intra-articular fractures often lead to post-traumatic arthritis. Research does not indicate a correlation between intra-articular fractures and FES, osteomyelitis, or compartment syndrome.

The patient scheduled for a Syme amputation is concerned about the ability to eventually stand on the amputated extremity. How should the nurse best respond to the patient's concern? A) "You will eventually be able to withstand full weight-bearing after the amputation." B) "You will have minimal weight-bearing on this extremity but you'll be taught how to use an assistive device." C) "You likely will not be able to use this extremity but you will receive teaching on use of a wheelchair." D) "You will be fitted for a prosthesis which may or may not allow you to walk."

Ans: A Feedback: Syme amputation (modified ankle disarticulation amputation) is performed most frequently for extensive foot trauma and produces a painless, durable extremity end that can withstand full weight-bearing. Therefore, each of the other teaching statements is incorrect.

A nurse is caring for a patient who had a right below-the-knee amputation (BKA). The nurse recognizes the importance of implementing measures that focus on preventing flexion contracture of the hip and maintaining proper positioning. Which of the following measures will best achieve these goals? A) Encouraging the patient to turn from side to side and to assume a prone position B) Initiating ROM exercises of the hip and knee 10 to 12 weeks after the amputation C) Minimizing movement of the flexor muscles of the hip D) Encouraging the patient to sit in a chair for at least 8 hours a day

Ans: A Feedback: The nurse encourages the patient to turn from side to side and to assume a prone position, if possible, to stretch the flexor muscles and to prevent flexion contracture of the hip. Postoperative ROM exercises are started early, because contracture deformities develop rapidly. ROM exercises include hip and knee exercises for patients with BKAs. The nurse also discourages sitting for prolonged periods of time.

A patient has returned to the postsurgical unit from the PACU after an above-the-knee amputation of the right leg. Results of the nurse's initial postsurgical assessment were unremarkable but the patient has called out. The nurse enters the room and observes copious quantities of blood at the surgical site. What should be the nurse's initial action? A) Apply a tourniquet. B) Elevate the residual limb. C) Apply sterile gauze. D) Call the surgeon.

Ans: A Feedback: The nurse should apply a tourniquet in the event of postsurgical hemorrhage. Elevating the limb and applying sterile gauze are likely insufficient to stop the hemorrhage. The nurse should attempt to control the immediate bleeding before contacting the surgeon.

A nurse is writing a care plan for a patient admitted to the emergency department (ED) with an open fracture. The nurse will assign priority to what nursing diagnosis for a patient with an open fracture of the radius? A) Risk for Infection B) Risk for Ineffective Role Performance C) Risk for Perioperative Positioning Injury D) Risk for Powerlessness

Ans: A Feedback: The patient has a significant risk for osteomyelitis and tetanus due to the fact that the fracture is open. Powerlessness and ineffective role performance are psychosocial diagnoses that may or may not apply, and which would be superseded by immediate physiologic threats such as infection. Surgical positioning injury is not plausible, since surgery is not likely indicated.

An emergency department nurse is assessing a 17-year-old soccer player who presented with a knee injury. The patient's description of the injury indicates that his knee was struck medially while his foot was on the ground. The nurse knows that the patient likely has experienced what injury? A) Lateral collateral ligament injury B) Medial collateral ligament injury C) Anterior cruciate ligament injury D) Posterior cruciate ligament injury

Ans: A Feedback: When the knee is struck medially, damage may occur to the lateral collateral ligament. If the knee is struck laterally, damage may occur to the medial collateral ligament. The ACL and PCL are not typically injured in this way.

A nurse is planning the care of an older adult patient who will soon be discharged home after treatment for a fractured hip. In an effort to prevent future fractures, the nurse should encourage which of the following? Select all that apply. A) Regular bone density testing B) A high-calcium diet C) Use of falls prevention precautions D) Use of corticosteroids as ordered E) Weight-bearing exercise

Ans: A, B, C, E Feedback: Health promotion measures after an older adult's hip fracture include weight-bearing exercise, promotion of a healthy diet, falls prevention, and bone density testing. Corticosteroids have the potential to reduce bone density and increase the risk for fractures.

An older adult patient experienced a fall and required treatment for a fractured hip on the orthopedic unit. Which of the following are contributory factors to the incidence of falls and fractured hips among the older adult population? Select all that apply. A) Loss of visual acuity B) Adverse medication effects C) Slowed reflexes D) Hearing loss E) Muscle weakness

Ans: A, B, C, E Feedback: Older adults are generally vulnerable to falls and have a high incidence of hip fracture. Weak quadriceps muscles, medication effects, vision loss, and slowed reflexes are among the factors that contribute to the incidence of falls. Decreased hearing is not noted to contribute to the incidence of falls.

A nurse admits a patient who has a fracture of the nose that has resulted in a skin tear and involvement of the mucous membranes of the nasal passages. The orthopedic nurse is aware that this description likely indicates which type of fracture? A) Compression B) Compound C) Impacted D) Transverse

Ans: B Feedback: A compound fracture involves damage to the skin or mucous membranes and is also called an open fracture. A compression fracture involves compression of bone and is seen in vertebral fractures. An impacted fracture occurs when a bone fragment is driven into another bone fragment. A transverse fracture occurs straight across the bone shaft.

An older adult patient has fallen in her home and is brought to the emergency department by ambulance with a suspected fractured hip. X-rays confirm a fracture of the left femoral neck. When planning assessments during the patient's presurgical care, the nurse should be aware of the patient's heightened risk of what complication? A) Osteomyelitis B) Avascular necrosis C) Phantom pain D) Septicemia

Ans: B Feedback: Fractures of the neck of the femur may damage the vascular system that supplies blood to the head and the neck of the femur, and the bone may become ischemic. For this reason, AVN is common in patients with femoral neck fractures. Infections are not immediate complications and phantom pain applies to patients with amputations, not hip fractures.

An elite high school football player has been diagnosed with a shoulder dislocation. The patient has been treated and is eager to resume his role on his team, stating that he is not experiencing pain. What should the nurse emphasize during health education? A) The need to take analgesia regardless of the short-term absence of pain B) The importance of adhering to the prescribed treatment and rehabilitation regimen C) The fact that he has a permanently increased risk of future shoulder dislocations D) The importance of monitoring for intracapsular bleeding once he resumes playing

Ans: B Feedback: Patients who have experienced sports-related injuries are often highly motivated to return to their previous level of activity. Adherence to restriction of activities and gradual resumption of activities needs to be reinforced. Appropriate analgesia use must be encouraged, but analgesia does not necessarily have to be taken in the absence of pain. If healing is complete, the patient does not likely have a greatly increased risk of reinjury. Dislocations rarely cause bleeding after the healing process.

A nurse is caring for a patient who has suffered an unstable thoracolumbar fracture. Which of the following is the priority during nursing care? A) Preventing infection B) Maintaining spinal alignment C) Maximizing function D) Preventing increased intracranial pressure

Ans: B Feedback: Patients with an unstable fracture must have their spine in alignment at all times in order to prevent neurologic damage. This is a greater threat, and higher priority, than promoting function and preventing infection, even though these are both valid considerations. Increased ICP is not a high risk.

Which of the following is the most appropriate nursing intervention to facilitate healing in a patient who has suffered a hip fracture? A) Administer analgesics as required. B) Place a pillow between the patient's legs when turning. C) Maintain prone positioning at all times. D) Encourage internal and external rotation of the affected leg.

Ans: B Feedback: Placing a pillow between the patient's legs when turning prevents adduction and supports the patient's legs. Administering analgesics addresses pain but does not directly protect bone remodeling and promote healing. Rotation of the affected leg can cause dislocation and must be avoided. Prone positioning does not need to be maintained at all times.

The orthopedic nurse should assess for signs and symptoms of Volkmann's contracture if a patient has fractured which of the following bones? A) Femur B) Humerus C) Radial head D) Clavicle

Ans: B Feedback: The most serious complication of a supracondylar fracture of the humerus is Volkmann's ischemic contracture, which results from antecubital swelling or damage to the brachial artery. This complication is specific to humeral fractures.

A patient has sustained a long bone fracture and the nurse is preparing the patient's care plan. Which of the following should the nurse include in the care plan? A) Administer vitamin D and calcium supplements as ordered. B) Monitor temperature and pulses of the affected extremity. C) Perform passive range of motion exercises as tolerated. D) Administer corticosteroids as ordered.

Ans: B Feedback: The nurse should include monitoring for sufficient blood supply by assessing the color, temperature, and pulses of the affected extremity. Weight-bearing exercises are encouraged, but passive ROM exercises have the potential to cause pain and inhibit healing. Corticosteroids, vitamin D, and calcium are not normally administered.

A patient with a simple arm fracture is receiving discharge education from the nurse. What would the nurse instruct the patient to do? A) Elevate the affected extremity to shoulder level when at rest. B) Engage in exercises that strengthen the unaffected muscles. C) Apply topical anesthetics to accessible skin surfaces as needed. D) Avoid using analgesics so that further damage is not masked.

Ans: B Feedback: The nurse will encourage the patient to engage in exercises that strengthen the unaffected muscles. Comfort measures may include appropriate use of analgesics and elevation of the affected extremity to the heart level. Topical anesthetics are not typically used.

The nurse is providing care for a patient who has had a below-the-knee amputation. The nurse enters the patient's room and finds him resting in bed with his residual limb supported on pillow. What is the nurse's most appropriate action? A) Inform the surgeon of this finding. B) Explain the risks of flexion contracture to the patient. C) Transfer the patient to a sitting position. D) Encourage the patient to perform active ROM exercises with the residual limb.

Ans: B Feedback: The residual limb should not be placed on a pillow, because a flexion contracture of the hip may result. There is no acute need to contact the patient's surgeon. Encouraging exercise or transferring the patient does not address the risk of flexion contracture.

A nurse's assessment of a patient's knee reveals edema, tenderness, muscle spasms, and ecchymosis. The patient states that 2 days ago he ran 10 miles and now it "really hurts to stand up." The nurse should plan care based on the belief that the patient has experienced what? A) A first-degree strain B) A second-degree strain C) A first-degree sprain D) A second-degree sprain

Ans: B Feedback: A second-degree strain involves tearing of muscle fibers and is manifested by notable loss of load-bearing strength with accompanying edema, tenderness, muscle spasm, and ecchymosis. A first-degree strain reflects tearing of a few muscle fibers and is accompanied by minor edema, tenderness, and mild muscle spasm, without noticeable loss of function. However, this patient states a loss of function. A sprain normally involves twisting, which is inconsistent with the patient's overuse injury.

A nurse in a busy emergency department provides care for many patients who present with contusions, strains, or sprains. Treatment modalities that are common to all of these musculoskeletal injuries include which of the following? Select all that apply. A) Massage B) Applying ice C) Compression dressings D) Resting the affected extremity E) Corticosteroids F) Elevating the injured limb

Ans: B, C, D, F Feedback: Treatment of contusions, strains, and sprains consists of resting and elevating the affected part, applying cold, and using a compression bandage. Massage and corticosteroids are not used to treat these injuries.

A school nurse is assessing a student who was kicked in the shin during a soccer game. The area of the injury has become swollen and discolored. The triage nurse recognizes that the patient has likely sustained what? A) Sprain B) Strain C) Contusion D) Dislocation

Ans: C Feedback: A contusion is a soft-tissue injury that results in bleeding into soft tissues, creating a hematoma and ecchymosis. A sprain is an injury to ligaments caused by wrenching or twisting. A strain is a "muscle pull" from overuse, overstretching, or excessive stress. A dislocation is a condition in which the articular surfaces of the bones forming a joint are no longer in anatomic contact. Because the injury is not at the site of a joint, the patient has not experienced a sprain, strain, or dislocation.

A nurse is planning the care of a patient with osteomyelitis that resulted from a diabetic foot ulcer. The patient requires a transmetatarsal amputation. When planning the patient's postoperative care, which of the following nursing diagnoses should the nurse most likely include in the plan of care? A) Ineffective Thermoregulation B) Risk-Prone Health Behavior C) Disturbed Body Image D) Deficient Diversion Activity

Ans: C Feedback: Amputations present a serious threat to any patient's body image. None of the other listed diagnoses is specifically associated with amputation.

A patient has presented to the emergency department with an injury to the wrist. The patient is diagnosed with a third-degree strain. Why would the physician order an x-ray of the wrist? A) Nerve damage is associated with third-degree strains. B) Compartment syndrome is associated with third-degree strains. C) Avulsion fractures are associated with third-degree strains. D) Greenstick fractures are associated with third-degree strains.

Ans: C Feedback: An x-ray should be obtained to rule out bone injury, because an avulsion fracture (in which a bone fragment is pulled away from the bone by a tendon) may be associated with a third-degree strain. Nerve damage, compartment syndrome, and greenstick fractures are not associated with third-degree strains.

A nurse is caring for a patient who has suffered a hip fracture and who will require an extended hospital stay. The nurse should ensure that the patient does which of the following in order to prevent common complications associated with a hip fracture? A) Avoid requesting analgesia unless pain becomes unbearable. B) Use supplementary oxygen when transferring or mobilizing. C) Increase fluid intake and perform prescribed foot exercises. D) Remain on bed rest for 14 days or until instructed by the orthopedic surgeon.

Ans: C Feedback: Deep vein thrombosis (DVT) is among the most common complications related to a hip fracture. To prevent DVT, the nurse encourages intake of fluids and ankle and foot exercises. The patient should not be told to endure pain; a proactive approach to pain control should be adopted. While respiratory complications commonly include atelectasis and pneumonia, the use of deep-breathing exercises, changes in position at least every 2 hours, and the use of incentive spirometry help prevent the respiratory complications more than using supplementary oxygen. Bed rest may be indicated in the short term, but is not normally required for 14 days.

A nurse is performing a shift assessment on an elderly patient who is recovering after surgery for a hip fracture. The nurse notes that the patient is complaining of chest pain, has an increased heart rate, and increased respiratory rate. The nurse further notes that the patient is febrile and hypoxic, coughing, and producing large amounts of thick, white sputum. The nurse recognizes that this is a medical emergency and calls for assistance, recognizing that this patient is likely demonstrating symptoms of what complication? A) Avascular necrosis of bone B) Compartment syndrome C) Fat embolism syndrome D) Complex regional pain syndrome

Ans: C Feedback: Fat embolism syndrome occurs most frequently in young adults and elderly patients who experience fractures of the proximal femur (i.e., hip fracture). Presenting features of fat embolism syndrome include hypoxia, tachypnea, tachycardia, and pyrexia. The respiratory distress response includes tachypnea, dyspnea, wheezes, precordial chest pain, cough, large amounts of thick, white sputum, and tachycardia. Avascular necrosis (AVN) occurs when the bone loses its blood supply and dies. This does not cause coughing. Complex regional pain syndrome does not have cardiopulmonary involvement.

A patient has come to the orthopedic clinic for a follow-up appointment 6 weeks after fracturing his ankle. Diagnostic imaging reveals that bone union is not taking place. What factor may have contributed to this complication? A) Inadequate vitamin D intake B) Bleeding at the injury site C) Inadequate immobilization D) Venous thromboembolism (VTE)

Ans: C Feedback: Inadequate fracture immobilization can delay or prevent union. A short-term vitamin D deficiency would not likely prevent bone union. VTE is a serious complication but would not be a cause of nonunion. Similarly, bleeding would not likely delay union.

A 20 year-old is brought in by ambulance to the emergency department after being involved in a motorcycle accident. The patient has an open fracture of his tibia. The wound is highly contaminated and there is extensive soft-tissue damage. How would this patient's fracture likely be graded? A) Grade I B) Grade II C) Grade III D) Grade IV

Ans: C Feedback: Open fractures are graded according to the following criteria. Grade I is a clean wound less than 1 cm long. Grade II is a larger wound without extensive soft-tissue damage. Grade III is highly contaminated, has extensive soft-tissue damage, and is the most severe. There is no grade IV fracture.

Six weeks after an above-the-knee amputation (AKA), a patient returns to the outpatient office for a routine postoperative checkup. During the nurse's assessment, the patient reports symptoms of phantom pain. What should the nurse tell the patient to do to reduce the discomfort of the phantom pain? A) Apply intermittent hot compresses to the area of the amputation. B) Avoid activity until the pain subsides. C) Take opioid analgesics as ordered. D) Elevate the level of the amputation site.

Ans: C Feedback: Opioid analgesics may be effective in relieving phantom pain. Heat, immobility, and elevation are not noted to relieve this form of pain.

A young patient is being treated for a femoral fracture suffered in a snowboarding accident. The nurse's most recent assessment reveals that the patient is uncharacteristically confused. What diagnostic test should be performed on this patient? A) Electrolyte assessment B) Electrocardiogram C) Arterial blood gases D) Abdominal ultrasound

Ans: C Feedback: Subtle personality changes, restlessness, irritability, or confusion in a patient who has sustained a fracture are indications for immediate arterial blood gas studies due to the possibility of fat embolism syndrome. This assessment finding does not indicate an immediate need for electrolyte levels, an ECG, or abdominal ultrasound.

A patient is being treated for a fractured hip and the nurse is aware of the need to implement interventions to prevent muscle wasting and other complications of immobility. What intervention best addresses the patient's need for exercise? A) Performing gentle leg lifts with both legs B) Performing massage to stimulate circulation C) Encouraging frequent use of the overbed trapeze D) Encouraging the patient to log roll side to side once per hour

Ans: C Feedback: The patient is encouraged to exercise as much as possible by means of the overbed trapeze. This device helps strengthen the arms and shoulders in preparation for protected ambulation. Independent logrolling may result in injury due to the location of the fracture. Leg lifts would be contraindicated for the same reason. Massage by the nurse is not a substitute for exercise.

Limbic system

Area of the brain located above the brain stem that includes the thalamus, hypothalamus, hippocampus, and amygdala

A patient is admitted to the orthopedic unit with a fractured femur after a motorcycle accident. The patient has been placed in traction until his femur can be rodded in surgery. For what early complications should the nurse monitor this patient? Select all that apply. A) Systemic infection B) Complex regional pain syndrome C) Deep vein thrombosis D) Compartment syndrome E) Fat embolism

Ans: C, D, E Feedback: Early complications include shock, fat embolism, compartment syndrome, and venous thromboemboli (deep vein thrombosis [DVT], pulmonary embolism [PE]). Infection and CRPS are later complications of fractures.

A nurse is preparing to discharge an emergency department patient who has been fitted with a sling to support her arm after a clavicle fracture. What should the nurse instruct the patient to do? A) Elevate the arm above the shoulder 3 to 4 times daily. B) Avoid moving the elbow, wrist, and fingers until bone remodeling is complete. C) Engage in active range of motion using the affected arm. D) Use the arm for light activities within the range of motion.

Ans: D Feedback: A patient with a clavicle fracture may use a sling to support the arm and relieve the pain. The patient may be permitted to use the arm for light activities within the range of comfort. The patient should not elevate the arm above the shoulder level until the ends of the bones have united, but the nurse should encourage the patient to exercise the elbow, wrist, and fingers.

Radiographs of a boy's upper arm show that the humerus appears to be fractured on one side and slightly bent on the other. This diagnostic result suggests what type of fracture? A) Impacted B) Compound C) Compression D) Greenstick

Ans: D Feedback: Greenstick fractures are an incomplete fracture that results in the bone being broken on one side, while the other side is bent. This is not characteristic of an impacted, compound, or compression fracture.

An emergency department patient is diagnosed with a hip dislocation. The patient's family is relieved that the patient has not suffered a hip fracture, but the nurse explains that this is still considered to be a medical emergency. What is the rationale for the nurse's statement? A) The longer the joint is displaced, the more difficult it is to get it back in place. B) The patient's pain will increase until the joint is realigned. C) Dislocation can become permanent if the process of bone remodeling begins. D) Avascular necrosis may develop at the site of the dislocation if it is not promptly resolved.

Ans: D Feedback: If a dislocation or subluxation is not reduced immediately, avascular necrosis (AVN) may develop. Bone remodeling does not take place because a fracture has not occurred. Realignment does not become more difficult with time and pain would subside with time, not become worse.

A patient is brought to the emergency department by ambulance after stepping in a hole and falling. While assessing him the nurse notes that his right leg is shorter than his left leg; his right hip is noticeably deformed and he is in acute pain. Imaging does not reveal a fracture. Which of the following is the most plausible explanation for this patient's signs and symptoms? A) Subluxated right hip B) Right hip contusion C) Hip strain D) Traumatic hip dislocation

Ans: D Feedback: Signs and symptoms of a traumatic dislocation include acute pain, change in positioning of the joint, shortening of the extremity, deformity, and decreased mobility. A subluxation would cause moderate deformity, or possibly no deformity. A contusion or strain would not cause obvious deformities.

A patient who has had an amputation is being cared for by a multidisciplinary rehabilitation team. What is the primary goal of this multidisciplinary team? A) Maximize the efficiency of care B) Ensure that the patient's health care is holistic C) Facilitate the patient's adjustment to a new body image D) Promote the patient's highest possible level of function

Ans: D Feedback: The multidisciplinary rehabilitation team helps the patient achieve the highest possible level of function and participation in life activities. The team is not primarily motivated by efficiency, the need for holistic care, or the need to foster the patient's body image, despite the fact that each of these are valid goals.

A nurse is preparing to discharge a patient from the emergency department after receiving treatment for an ankle sprain. While providing discharge education, the nurse should encourage which of the following? A) Apply heat for the first 24 to 48 hours after the injury. B) Maintain the ankle in a dependent position. C) Exercise hourly by performing rotation exercises of the ankle. D) Keep an elastic compression bandage on the ankle.

Ans: D Feedback: Treatment of a sprain consists of resting and elevating the affected part, applying cold, and using a compression bandage. After the acute inflammatory stage (usually 24 to 48 hours after injury), heat may be applied intermittently. Rotation exercises would likely be painful.

Corticosteroids (Glucocorticoids)

Antiinflammatory properties Used for chronic asthma Do not relieve symptoms of acute asthma attacks May be administered IV Oral or inhaled forms -Inhaled forms reduce systemic effects May take several weeks before full effects are seen

Psychotropic drugs

Antipsychotics, antidepressants, mood stabilizers, anxiolytics, and stimulants

When completing a physical assessment of an individual's response to stress, the nurse should observe and inquire about what?

Appetite and sleep

The nurse notes that an older adult client is wearing layers of clothing on a warm, fall day. Which would be the priority assessment at this time?

Asking whether the client often feels cold Dress is typically appropriate for occasion and weather, and dress varies considerably from person to person. Some older adults may wear excess clothing because of slowed metabolism and loss of subcutaneous fat resulting in cold intolerance. The nurse needs to determine this first before performing any other assessments.

Occipital lobe

Assists in visual interpretation and coordinating language generation.

Methylphenidate (Ritalin)

Avoid in emotionally unstable clients, especially if they have a dependence on drugs and alcohol as they may increase dosage on their own. Chronic abuse can lead to dependence.

A nurse is caring for a patient who is being treated in the hospital for a spontaneous vertebral fracture related to osteoporosis. The nurse should address the nursing diagnosis of Acute Pain Related to Fracture by implementing what intervention? A) Maintenance of high Fowlers positioning whenever possible B) Intermittent application of heat to the patients back C) Use of a pressure-reducing mattress D) Passive range of motion exercises

B

A patient has been admitted to the hospital with a spontaneous vertebral fracture related to osteoporosis. Which of the following nursing diagnoses must be addressed in the plan of care? A) Risk for Aspiration Related to Vertebral Fracture B) Constipation Related to Vertebral Fracture C) Impaired Swallowing Related to Vertebral Fracture D) Decreased Cardiac Output Related to Vertebral Fracture

B

A patient has come to the clinic for a routine annual physical. The nurse practitioner notes a palpable, painless projection of bone at the patients shoulder. The projection appears to be at the distal end of the humerus. The nurse should suspect the presence of which of the following? A) Osteomyelitis B) Osteochondroma C) Osteomalacia D) Pagets disease

B

A patient has returned to the unit after undergoing limb-sparing surgery to remove a metastatic bone tumor. The nurse providing postoperative care in the days following surgery assesses for what complication from surgery? A) Deficient fluid volume B) Delayed wound healing C) Hypocalcemia D) Pathologic fractures

B

A patient presents at a clinic complaining of pain in his heel so bad that it inhibits his ability to walk. The patient is subsequently diagnosed with plantar fasciitis. This patients plan of care should include what intervention? A) Wrapping the affected area in lambs wool or gauze to relieve pressure B) Gently stretching the foot and the Achilles tendon C) Wearing open-toed shoes at all times D) Applying topical analgesic ointment to plantar surface each morning

B

A patient with diabetes has been diagnosed with osteomyelitis. The nurse notes that the patients right foot is pale and mottled, cool to touch, with a capillary refill of greater than 3 seconds. The nurse should suspect what type of osteomyelitis? A) Hematogenous osteomyelitis B) Osteomyelitis with vascular insufficiency C) Contiguous-focus osteomyelitis D) Osteomyelitis with muscular deterioration

B

A patient with diabetes is attending a class on the prevention of associated diseases. What action should the patient perform to reduce the risk of osteomyelitis? A) Increase calcium and vitamin intake. B) Perform meticulous foot care. C) Exercise 3 to 4 times weekly for at least 30 minutes. D) Take corticosteroids as ordered.

B

Which of the following patients should the nurse recognize as being at the highest risk for the development of osteomyelitis? A) A middle-age adult who takes ibuprofen daily for rheumatoid arthritis B) An elderly patient with an infected pressure ulcer in the sacral area C) A 17-year-old football player who had orthopedic surgery 6 weeks prior D) An infant diagnosed with jaundice

B

15. You are the nurse planning an educational event for the nurses on a subacute medical unit on the topic of normal, age-related physiological changes. What phenomenon would you include in your teaching plan? A) A decrease in cognition, judgment, and memory B) A decrease in muscle mass and bone density C) The disappearance of sexual desire for both men and women D) An increase in sebaceous and sweat gland function in both men and women

B) A decrease in muscle mass and bone density

34. A school nurse is called to the playground where a 6-year-old girl has been found unresponsive and staring into space, according to the playground supervisor. How would the nurse document the girls activity in her chart at school? A) Generalized seizure B) Absence seizure C) Focal seizure D) Unclassified seizure

B) Absence seizure

A patient with herpes simplex virus encephalitis (HSV) has been admitted to the ICU. What medication would the nurse expect the physician to order for the treatment of this disease process? A) Cyclosporine (Neoral) B) Acyclovir (Zovirax) C) Cyclobenzaprine (Flexeril) D) Ampicillin (Prinicpen)

B) Acyclovir (Zovirax)

13. The admissions department at a local hospital is registering an elderly man for an outpatient diagnostic test. The admissions nurse asks the man if he has an advanced directive. The man responds that he does not want to complete an advance directive because he does not want anyone controlling his finances. What would be appropriate information for the nurse to share with this patient? A) Advance directives are not legal documents, so you have nothing to worry about. B) Advance directives are limited only to health care instructions and directives. C) Your finances cannot be managed without an advance directive. D) Advance directives are implemented when you become incapacitated, and then you will use a living will to allow the state to manage your money.

B) Advance directives are limited only to health care instructions and directives.

3. An occupational health nurse overhears an employee talking to his manager about a 65-year-old coworker. What phenomenon would the nurse identify when hearing the employee state, He should just retire and make way for some new blood.? A) Intolerance B) Ageism C) Dependence D) Nonspecific prejudice

B) Ageism

16. A clinic nurse is caring for a patient diagnosed with migraine headaches. During the patient teaching session, the patient questions the nurse regarding alcohol consumption. What would the nurse be correct in telling the patient about the effects of alcohol? A) Alcohol causes hormone fluctuations. B) Alcohol causes vasodilation of the blood vessels. C) Alcohol has an excitatory effect on the CNS. D) Alcohol diminishes endorphins in the brain.

B) Alcohol causes vasodilation of the blood vessels.

A gerontologic nurse is making an effort to address some of the misconceptions about older adults that exist among health care providers. The nurse has made the point that most people aged 75 years remains functionally independent. The nurse should attribute this trend to what factor? A) Early detection of disease and increased advocacy by older adults B) Application of health-promotion and disease-prevention activities C) Changes in the medical treatment of hypertension and hyperlipidemia D) Genetic changes that have resulted in increased resiliency to acute infection

B) Application of health-promotion and disease-prevention activities

28. A gerontologic nurse has been working hard to change the perceptions of the elderly, many of which are negative, by other segments of the population. What negative perceptions of older people have been identified in the literature? Select all that apply. A) As being the cause of social problems B) As not contributing to society C) As draining economic resources D) As competing with children for resources E) As dominating health care research

B) As not contributing to society C) As draining economic resources D) As competing with children for resources

38. A nurse is caring for a patient who experiences debilitating cluster headaches. The patient should be taught to take appropriate medications at what point in the course of the onset of a new headache? A) As soon as the patients pain becomes unbearable B) As soon as the patient senses the onset of symptoms C) Twenty to 30 minutes after the onset of symptoms D) When the patient senses his or her symptoms peaking

B) As soon as the patient senses the onset of symptoms

To alleviate pain associated with trigeminal neuralgia, a patient is taking Tegretol (carbamazepine). What health education should the nurse provide to the patient before initiating this treatment? A) Concurrent use of calcium supplements is contraindicated. B) Blood levels of the drug must be monitored. C) The drug is likely to cause hyperactivity and agitation. D) Tegretol can cause tinnitus during the first few days of treatment.

B) Blood levels of the drug must be monitored.

10. While completing a health history on a patient who has recently experienced a seizure, the nurse would assess for what characteristic associated with the postictal state? A) Epileptic cry B) Confusion C) Urinary incontinence D) Body rigidity

B) Confusion

A patient diagnosed with MS has been admitted to the medical unit for treatment of an MS exacerbation. Included in the admission orders is baclofen (Lioresal). What should the nurse identify as an expected outcome of this treatment? A) Reduction in the appearance of new lesions on the MRI B) Decreased muscle spasms in the lower extremities C) Increased muscle strength in the upper extremities D) Decreased severity and duration of exacerbations

B) Decreased muscle spasms in the lower extremities

A patient diagnosed with myasthenia gravis has been hospitalized to receive plasmapheresis for a myasthenic exacerbation. The nurse knows that the course of treatment for plasmapheresis in a patient with myasthenia gravis is what? A) Every day for 1 week B) Determined by the patient's response C) Alternate days for 10 days D) Determined by the patient's weight

B) Determined by the patient's response

10. The case manager is working with an 84-year-old patient newly admitted to a rehabilitation facility. When developing a care plan for this older adult, which factors should the nurse identify as positive attributes that benefit coping in this age group? Select all that apply. A) Decreased risk taking B) Effective adaptation skills C) Avoiding participation in untested roles D) Increased life experience E) Resiliency during change

B) Effective adaptation skills D) Increased life experience E) Resiliency during change

The nurse is developing a plan of care for a patient newly diagnosed with Bell's palsy. The nurse's plan of care should address what characteristic manifestation of this disease? A) Tinnitus B) Facial paralysis C) Pain at the base of the tongue D) Diplopia

B) Facial paralysis

20. A 47-year-old patient who has come to the physicians office for his annual physical is being assessed by the office nurse. The nurse who is performing routine health screening for this patient should be aware that one of the first physical signs of aging is what? A) Having more frequent aches and pains B) Failing eyesight, especially close vision C) Increasing loss of muscle tone D) Accepting limitations while developing assets

B) Failing eyesight, especially close vision

A patient exhibiting an altered level of consciousness (LOC) due to blunt-force trauma to the head is admitted to the ED. The physician determines the patients injury is causing increased intracranial pressure (ICP). The nurse should gauge the patients LOC on the results of what diagnostic tool? A) Monro-Kellie hypothesis B) Glasgow Coma Scale C) Cranial nerve function D) Mental status examination

B) Glasgow Coma Scale

A 69-year-old patient is brought to the ED by ambulance because a family member found him lying on the floor disoriented and lethargic. The physician suspects bacterial meningitis and admits the patient to the ICU. The nurse knows that risk factors for an unfavorable outcome include what? Select all that apply. A) Blood pressure greater than 140/90 mm Hg B) Heart rate greater than 120 bpm C) Older age D) Low Glasgow Coma Scale E) Lack of previous immunizations

B) Heart rate greater than 120 bpm C) Older age D) Low Glasgow Coma Scale

31. Gerontologic nursing is a specialty area of nursing that provides care for the elderly in our population. What goal of care should a gerontologic nurse prioritize when working with this population? A) Helping older adults determine how to reduce their use of external resources B) Helping older adults use their strengths to optimize independence C) Helping older adults promote social integration D) Helping older adults identify the weaknesses that most limit them

B) Helping older adults use their strengths to optimize independence

39. Older people have many altered reactions to disease that are based on age-related physiological changes. When the nurse observes physical indicators of illness in the older population, that nurse must remember which of the following principles? A) Potential life-threatening problems in the older adult population are not as serious as they are in a middle-aged population. B) Indicators that are useful and reliable in younger populations cannot be relied on as indications of potential life-threatening problems in older adults. C) The same physiological processes that indicate serious health care problems in a younger population indicate mild disease states in the elderly. D) Middle-aged people do not react to disease states the same as a younger population does.

B) Indicators that are useful and reliable in younger populations cannot be relied on as indications of potential life-threatening problems in older adults.

13. The nurse is caring for a patient who is in status epilepticus. What medication does the nurse know may be given to halt the seizure immediately? A) Intravenous phenobarbital (Luminal) B) Intravenous diazepam (Valium) C) Oral lorazepam (Ativan) D) Oral phenytoin (Dilantin)

B) Intravenous diazepam (Valium)

1. You are providing care for an 82-year-old man whose signs and symptoms of Parkinson disease have become more severe over the past several months. The man tells you that he can no longer do as many things for himself as he used to be able to do. What factor should you recognize as impacting your patients life most significantly? A) Neurologic deficits B) Loss of independence C) Age-related changes D) Tremors and decreased mobility

B) Loss of independence

12. The nurse is participating in the care of a patient with increased ICP. What diagnostic test is contraindicated in this patients treatment? A) Computed tomography (CT) scan B) Lumbar puncture C) Magnetic resonance imaging (MRI) D) Venous Doppler studies

B) Lumbar puncture

15. The neurologic ICU nurse is admitting a patient following a craniotomy using the supratentorial approach. How should the nurse best position the patient? A) Position the patient supine. B) Maintain head of bed (HOB) elevated at 30 to 45 degrees. C) Position patient in prone position. D) Maintain bed in Trendelenberg position.

B) Maintain head of bed (HOB) elevated at 30 to 45 degrees.

2. The nurse is providing care for a patient who is unconscious. What nursing intervention takes highest priority? A) Maintaining accurate records of intake and output B) Maintaining a patent airway C) Inserting a nasogastric (NG) tube as ordered D) Providing appropriate pain control

B) Maintaining a patent airway

4. The nurse is caring for a patient who is postoperative following a craniotomy. When writing the plan of care, the nurse identifies a diagnosis of deficient fluid volume related to fluid restriction and osmotic diuretic use. What would be an appropriate intervention for this diagnosis? A) Change the patients position as indicated. B) Monitor serum electrolytes. C) Maintain NPO status. D) Monitor arterial blood gas (ABG) values.

B) Monitor serum electrolytes.

A patient with possible bacterial meningitis is admitted to the ICU. What assessment finding would the nurse expect for a patient with this diagnosis? A) Pain upon ankle dorsiflexion of the foot B) Neck flexion produces flexion of knees and hips C) Inability to stand with eyes closed and arms extended without swaying D) Numbness and tingling in the lower extremities

B) Neck flexion produces flexion of knees and hips

A patient with MS has developed dysphagia as a result of cranial nerve dysfunction. What nursing action should the nurse consequently perform? A) Arrange for the patient to receive a low residue diet. B) Position the patient upright during feeding. C) Suction the patient following each meal. D) Withhold liquids until the patient has finished eating.

B) Position the patient upright during feeding.

A male patient presents to the clinic complaining of a headache. The nurse notes that the patient is guarding his neck and tells the nurse that he has stiffness in the neck area. The nurse suspects the patient may have meningitis. What is another well-recognized sign of this infection? A) Negative Brudzinski's sign B) Positive Kernig's sign C) Hyperpatellar reflex D) Sluggish pupil reaction

B) Positive Kernig's sign

The nurse is caring for a patient with multiple sclerosis (MS). The patient tells the nurse the hardest thing to deal with is the fatigue. When teaching the patient how to reduce fatigue, what action should the nurse suggest? A) Taking a hot bath at least once daily B) Resting in an air-conditioned room whenever possible C) Increasing the dose of muscle relaxants D) Avoiding naps during the day

B) Resting in an air-conditioned room whenever possible

27. Mrs. Harris is an 83-year-old woman who has returned to the community following knee replacement surgery. The community health nurse recognizes that Mrs. Harris has prescriptions for nine different medications for the treatment of varied health problems. In addition, she has experienced occasional episodes of dizziness and lightheadedness since her discharge. The nurse should identify which of the following nursing diagnoses? A) Risk for infection related to polypharmacy and hypotension B) Risk for falls related to polypharmacy and impaired balance C) Adult failure to thrive related to chronic disease and circulatory disturbance D) Disturbed thought processes related to adverse drug effects and hypotension

B) Risk for falls related to polypharmacy and impaired balance

35. You are the nurse caring for an elderly patient with cardiovascular disease. The patient comes to the clinic with a suspected respiratory infection and is diagnosed with pneumonia. As the nurse, what do you know about the altered responses of older adults? A) Treatments for older adults need to be more holistic than treatments used in the younger population. B) The altered responses of older adults reinforce the need for the nurse to monitor all body systems to identify possible systemic complications. C) The altered responses of older adults define the nursing interactions with the patient. D) Older adults become hypersensitive to antibiotic treatments for infectious disease states.

B) The altered responses of older adults reinforce the need for the nurse to monitor all body systems to identify possible systemic complications.

19. You are caring for a patient with late-stage Alzheimers disease. The patients wife tells you that the patient has now become completely dependent and that she feels guilty if she takes any time for herself. What outcomes would be appropriate for the nurse to develop to assist the patients wife? A) The caregiver learns to explain to the patient why she needs time for herself. B) The caregiver distinguishes essential obligations from those that can be controlled or limited. C) The caregiver leaves the patient at home alone for short periods of time to encourage independence. D) The caregiver prioritizes her own health over that of the patient.

B) The caregiver distinguishes essential obligations from those that can be controlled or limited.

A patient with metastatic cancer has developed trigeminal neuralgia and is taking carbamazepine (Tegretol) for pain relief. What principle applies to the administration of this medication? A) Tegretol is not known to have serious adverse effects. B) The patient should be monitored for bone marrow depression. C) Side effects of the medication include renal dysfunction. D) The medication should be first taken in the maximum dosage form to be effective.

B) The patient should be monitored for bone marrow depression.

Client teaching for mood stabilizers

Blood levels should be done periodically. Dose should be given on time so plasma levels are accurate when checked 12 hours after last dose. Take medication with meals to minimize nausea. No driving till dizziness, lethargy, fatigue, or blurred vision has subsided.

A nurse is caring for a patient with Pagets disease and is reviewing the patients most recent laboratory values. Which of the following values is most characteristic of Pagets disease? A) An elevated level of parathyroid hormone and low calcitonin levels B) A low serum alkaline phosphatase level and a low serum calcium level C) An elevated serum alkaline phosphatase level and a normal serum calcium level D) An elevated calcitonin level and low levels of parathyroid hormone

C

A nurse is caring for a patient with a bone tumor. The nurse is providing education to help the patient reduce the risk for pathologic fractures. What should the nurse teach the patient? A) Strive to achieve maximum weight-bearing capabilities. B) Gradually strengthen the affected muscles through weight training. C) Support the affected extremity with external supports such as splints. D) Limit reliance on assistive devices in order to build strength.

C

A nurse is discussing conservative management of tendonitis with a patient. Which of the following may be an effective approach to managing tendonitis? A) Weight reduction B) Use of oral opioid analgesics C) Intermittent application of ice and heat D) Passive range of motion exercises

C

A nurse is providing care for a patient who has osteomalacia. What major goal will guide the choice of medical and nursing interventions? A) Maintenance of skin integrity B) Prevention of bone metastasis C) Maintenance of adequate levels of activated vitamin D D) Maintenance of adequate parathyroid hormone function

C

A nurse is reviewing the pathophysiology that may underlie a patients decreased bone density. What hormone should the nurse identify as inhibiting bone resorption and promoting bone formation? A) Estrogen B) Parathyroid hormone (PTH) C) Calcitonin D) Progesterone

C

A nursing educator is reviewing the risk factors for osteoporosis with a group of recent graduates. What risk factor of the following should the educator describe? A) Recurrent infections and prolonged use of NSAIDs B) High alcohol intake and low body mass index C) Small frame, female gender, and Caucasian ethnicity D) Male gender, diabetes, and high protein intake

C

A patient presents at a clinic complaining of back pain that goes all the way down the back of the leg to the foot. The nurse should document the presence of what type of pain? A) Bursitis B) Radiculopathy C) Sciatica D) Tendonitis

C

Neurotransmitters

Chemicals that transmit information from one neuron to another known as neurotransmission. They can excite cells (excitatory) or inhibit cells (inhibitory). After the neurotransmitter has been released into the synapse and the relays the message to the receptor cells, they are either transported back to the synapse to the axon and stored for later use (re-uptake) or is metabolized and inactivated by enzymes (primarily monoamine oxidase).

The nurse is discharging a patient home after surgery for trigeminal neuralgia. What advice should the nurse provide to this patient in order to reduce the risk of injury? A) Avoid watching television or using a computer for more than 1 hour at a time. B) Use OTC antibiotic eye drops for at least 14 days. C) Avoid rubbing the eye on the affected side of the face. D) Rinse the eye on the affected side with normal saline daily for 1 week.

C) Avoid rubbing the eye on the affected side of the face.

Cyclic compounds

Cause varying degrees of sedation, orthostatic hypotension, anticholinergic side effects, and lethal on overdose.

16. A home health nurse makes a home visit to a 90-year-old patient who has cardiovascular disease. During the visit the nurse observes that the patient has begun exhibiting subtle and unprecedented signs of confusion and agitation. What should the home health nurse do? A) Increase the frequency of the patients home care. B) Have a family member check in on the patient in the evening. C) Arrange for the patient to see his primary care physician. D) Refer the patient to an adult day program.

C) Arrange for the patient to see his primary care physician.

Cerebellum

Center for coordinating movements and postural adjustments. Receives and integrates information from all of the body. Inhibition transmission of dopamine in this area is associated with the lack of smooth muscle coordinated movements in diseases (i.e. Parkinson's)

35. A neurologic nurse is reviewing seizures with a group of staff nurses. How should this nurse best describe the cause of a seizure? A) Sudden electrolyte changes throughout the brain B) A dysrhythmia in the peripheral nervous system C) A dysrhythmia in the nerve cells in one section of the brain D) Sudden disruptions in the blood flow throughout the brain

C) A dysrhythmia in the nerve cells in one section of the brain

40. You are the nurse caring for a 91-year-old patient admitted to the hospital for a fall. The patient complains of urge incontinence and tells you he most often falls when he tries to get to the bathroom in his home. You identify the nursing diagnosis of risk for falls related to impaired mobility and urinary incontinence. The older adults risk for falls is considered to be which of the following? A) The result of impaired cognitive functioning B) The accumulation of environmental hazards C) A geriatric syndrome D) An age-related health deficit

C) A geriatric syndrome

21. A gerontologic nurse is aware of the demographic changes that are occurring in the United States, and this affects the way that the nurse plans and provides care. Which of the following phenomena is currently undergoing the most rapid and profound change? A) More families are having to provide care for their aging members. B) Adult children find themselves participating in chronic disease management. C) A growing number of people live to a very old age. D) Elderly people are having more accidents, increasing the costs of health care.

C) A growing number of people live to a very old age.

A patient who has been on long-term phenytoin (Dilantin) therapy is admitted to the unit. In light of the adverse of effects of this medication, the nurse should prioritize which of the following in the patients plan of care? A) Monitoring of pulse oximetry B) Administration of a low-protein diet C) Administration of thorough oral hygiene D) Fluid restriction as ordered

C) Administration of thorough oral hygiene

Medulla

Center for respiration and cardiovascular functions.

Temporal lobe

Center for smell, hearing, memory, and emotional expression.

The brain consists of the

Cerebrum Cerebellum Brain stem Limbic system

A 33-year-old patient presents at the clinic with complaints of weakness, incoordination, dizziness, and loss of balance. The patient is hospitalized and diagnosed with MS. What sign or symptom, revealed during the initial assessment, is typical of MS? A) Diplopia, history of increased fatigue, and decreased or absent deep tendon reflexes B) Flexor spasm, clonus, and negative Babinski's reflex C) Blurred vision, intention tremor, and urinary hesitancy D) Hyperactive abdominal reflexes and history of unsteady gait and episodic paresthesia in both legs

C) Blurred vision, intention tremor, and urinary hesitancy

A patient is admitted through the ED with suspected St. Louis encephalitis. The unique clinical feature of St. Louis encephalitis will make what nursing action a priority? A) Serial assessments of hemoglobin levels B) Blood glucose monitoring C) Close monitoring of fluid balance D) Assessment of pain along dermatomes

C) Close monitoring of fluid balance

36. You are the nurse caring for patients in the urology clinic. A new patient, 78 years old, presents with complaints of urinary incontinence. An anticholinergic is prescribed. Why might this type of medication be an inappropriate choice in the elderly population? A) Gastrointestinal hypermotility can be an adverse effect of this medication. B) Detrusor instability can be an adverse effect of this medication. C) Confusion can be an adverse effect of this medication. D) Increased symptoms of urge incontinence can be an adverse effect of this medication.

C) Confusion can be an adverse effect of this medication.

An 84-year-old patient has returned from the post-anesthetic care unit (PACU) following hip arthroplasty. The patient is oriented to name only. The patient's family is very upset because, before having surgery, the patient had no cognitive deficits. The patient is subsequently diagnosed with postoperative delirium. What should the nurse explain to the patient's family? A) This problem is self-limiting and there is nothing to worry about. B) Delirium involves a progressive decline in memory loss and overall cognitive function. C) Delirium of this type is treatable and her cognition will return to previous levels. D) This problem can be resolved by administering antidotes to the anesthetic that was used in surgery.

C) Delirium of this type is treatable and her cognition will return to previous levels.

The nurse is caring for a patient with a brain tumor. What drug would the nurse expect to be ordered to reduce the edema surrounding the tumor? A) Solumedrol B) Dextromethorphan C) Dexamethasone D) Furosemide

C) Dexamethasone

A patient is being admitted to the neurologic ICU with suspected herpes simplex virus encephalitis. What nursing action best addresses the patient's complaints of headache? A) Initiating a patient-controlled analgesia (PCA) of morphine sulfate B) Administering hydromorphone (Dilaudid) IV as needed C) Dimming the lights and reducing stimulation D) Distracting the patient with activity

C) Dimming the lights and reducing stimulation

28. A patient has experienced a seizure in which she became rigid and then experienced alternating muscle relaxation and contraction. What type of seizure does the nurse recognize? A) Unclassified seizure B) Absence seizure C) Generalized seizure D) Focal seizure

C) Generalized seizure

22. As the population of the United States ages, research has shown that this aging will occur across all racial and ethnic groups. A community health nurse is planning an initiative that will focus on the group in which the aging population is expected to rise the fastest. What group should the nurse identify? A) Asian-Americans B) White non-Hispanics C) Hispanics D) African-Americans

C) Hispanics

A nurse is planning the care of a 28-year-old woman hospitalized with a diagnosis of myasthenia gravis. What approach would be most appropriate for the care and scheduling of diagnostic procedures for this patient? A) All at one time, to provide a longer rest period B) Before meals, to stimulate her appetite C) In the morning, with frequent rest periods D) Before bedtime, to promote rest

C) In the morning, with frequent rest periods

32. The presence of a gerontologic advanced practice nurse in a long-term care facility has proved beneficial to both the patients and the larger community in which they live. Nurses in this advanced practice role have been shown to cause what outcome? A) Greater interaction between younger adults and older adults occurs. B) The elderly recover more quickly from acute illnesses. C) Less deterioration takes place in the overall health of patients. D) The elderly are happier in long-term care facilities than at home.

C) Less deterioration takes place in the overall health of patients.

1. A patient is being admitted to the neurologic ICU following an acute head injury that has resulted in cerebral edema. When planning this patients care, the nurse would expect to administer what priority medication? A) Hydrochlorothiazide (HydroDIURIL) B) Furosemide (Lasix) C) Mannitol (Osmitrol) D) Spirolactone (Aldactone)

C) Mannitol (Osmitrol)

11. A patient with increased ICP has a ventriculostomy for monitoring ICP. The nurses most recent assessment reveals that the patient is now exhibiting nuchal rigidity and photophobia. The nurse would be correct in suspecting the presence of what complication? A) Encephalitis B) CSF leak C) Meningitis D) Catheter occlusion

C) Meningitis

The critical care nurse is caring for 25-year-old man admitted to the ICU with a brain abscess. What is a priority nursing responsibility in the care of this patient? A) Maintaining the patient's functional independence B) Providing health education C) Monitoring neurologic status closely D) Promoting mobility

C) Monitoring neurologic status closely

An 83-year-old woman was diagnosed with Alzheimers disease 2 years ago and the disease has progressed at an increasing pace in recent months. The patient has lost 16 pounds over the past 3 months, leading to a nursing diagnosis of Imbalanced Nutrition: Less than Body Requirements. What intervention should the nurse include in this patients plan of care? A) Offer the patient rewards for finishing all the food on her tray. B) Offer the patient bland, low-salt foods to limit offensiveness. C) Offer the patient only one food item at a time to promote focused eating. D) Arrange for insertion of a gastrostomy tube and initiate enteral feeding.

C) Offer the patient only one food item at a time to promote focused eating.

Neurotransmitter

Chemical substances manufactured in the neuron that aid in the transmission of information throughout the body and can be excitatory to inhibitory

3. The nurse is caring for a patient in the ICU who has a brain stem herniation and who is exhibiting an altered level of consciousness. Monitoring reveals that the patients mean arterial pressure (MAP) is 60 mm Hg with an intracranial pressure (ICP) reading of 5 mm Hg. What is the nurses most appropriate action? A) Position the patient in the high Fowlers position as tolerated. B) Administer osmotic diuretics as ordered. C) Participate in interventions to increase cerebral perfusion pressure. D) Prepare the patient for craniotomy.

C) Participate in interventions to increase cerebral perfusion pressure.

32. A patient is recovering from intracranial surgery that was performed using the transsphenoidal approach. The nurse should be aware that the patient may have required surgery on what neurologic structure? A) Cerebellum B) Hypothalamus C) Pituitary gland D) Pineal gland

C) Pituitary gland

A patient with Guillain-Barré syndrome has experienced a sharp decline in vital capacity. What is the nurse's most appropriate action? A) Administer bronchodilators as ordered. B) Remind the patient of the importance of deep breathing and coughing exercises. C) Prepare to assist with intubation. D) Administer supplementary oxygen by nasal cannula.

C) Prepare to assist with intubation.

The critical care nurse is admitting a patient in myasthenic crisis to the ICU. The nurse should prioritize what nursing action in the immediate care of this patient? A) Suctioning secretions B) Facilitating ABG analysis C) Providing ventilatory assistance D) Administering tube feedings

C) Providing ventilatory assistance

After a sudden decline in cognition, a 77-year-old man who has been diagnosed with vascular dementia is receiving care in his home. To reduce this mans risk of future infarcts, what action should the nurse most strongly encourage? A) Activity limitation and falls reduction efforts B) Adequate nutrition and fluid intake C) Rigorous control of the patients blood pressure and serum lipid levels D) Use of mobility aids to promote independence

C) Rigorous control of the patients blood pressure and serum lipid levels

A nurse is admitting a patient with a severe migraine headache and a history of acute coronary syndrome. What migraine medication would the nurse question for this patient? A) Rizatriptan (Maxalt) B) Naratriptan (Amerge) C) Sumatriptan succinate (Imitrex) D) Zolmitriptan (Zomig)

C) Sumatriptan succinate (Imitrex)

20. Following a traumatic brain injury, a patient has been in a coma for several days. Which of the following statements is true of this patients current LOC? A) The patient occasionally makes incomprehensible sounds. B) The patients current LOC will likely become a permanent state. C) The patient may occasionally make nonpurposeful movements. D) The patient is incapable of spontaneous respirations.

C) The patient may occasionally make nonpurposeful movements.

A 48-year-old patient has been diagnosed with trigeminal neuralgia following recent episodes of unilateral face pain. The nurse should recognize what implication of this diagnosis? A) The patient will likely require lifelong treatment with anticholinergic medications. B) The patient has a disproportionate risk of developing myasthenia gravis later in life. C) The patient needs to be assessed for MS. D) The disease is self-limiting and the patient will achieve pain relief over time.

C) The patient needs to be assessed for MS.

39. A nurse is collaborating with the interdisciplinary team to help manage a patients recurrent headaches. What aspect of the patients health history should the nurse identify as a potential contributor to the patients headaches? A) The patient leads a sedentary lifestyle. B) The patient takes vitamin D and calcium supplements. C) The patient takes vasodilators for the treatment of angina. D) The patient has a pattern of weight loss followed by weight gain.

C) The patient takes vasodilators for the treatment of angina.

Carbamazepine

Can cause aplastic anemia and agranulocytosis. Hematologic baseline must be obtained before initiating treatment, should be repeated throughout to check for low WBC and platelets.

Valproic Acid (Depakote)

Can cause hepatic failure, liver function tests should be done before therapy starts and at frequent intervals thereafter for the first 6 months. Can produce teratogenic effects. Can cause life threatening pancreatitis.

Thioridazine (Mellaril), Droperidol (Inapsine), and Mesoridazine (Serentil)

Can cause lengthening of the QT interval, a QT interval longer than 500ms is life threatening and can cause dysarrythmias, torsade de pointes, and sudden death.

Premoline (Cylert)

Can cause life-threatening liver failure within 4 weeks of onset. Must obtain written consent before initiating therapy.

Serotonin Syndrome

Can result front taking MAOIs and SSRIs at the same time. S/Sx: agitation, sweating, fever, tachycardia, hypotension, rigidity, hyerreflexia, coma, death

The nurse is assessing a client with psychiatric disorder. The nurse finds that when asked a question, the client gives excessive and unnecessary details followed by the answer. This is indicative of which impairment of thought content?

Circumstantial thinking When a client gives excessive and unnecessary details and then gives the answer, this is termed circumstantial thinking. Flight of ideas is characterized by an excessive amount and rate of speech composed of fragmented or unrelated ideas. Loose association is characterized by jumpimg from one idea to another with little or no evident relation between the thoughts. Thought broadcasting is when the client has the delusional belief that others can hear or know what the client is thinking.

Which must be addressed to establish a trusting working relationship before proceeding with the assessment?

Client's feelings and perceptions

The nurse is preparing a psychosocial assessment for use with clients with various mental health conditions. For which group of clients should the nurse include mostly closed-ended questions?

Clients with adult attention deficit hyperactivity disorder

A nurse is assessing a hospitalized client who is hearing voices due to psychosis. The client is easily distracted, and this is creating a barrier to completing the assessment. What is the most effective way for the nurse to proceed?

Complete the assessment in several short interactions.

Frontal lobe

Controls organization of thought, body movements, memories, emotions, and moral behaviour. Integration of all this information regulates arousal, focuses attention, and enables problem solving, and decision making. Abnormalities in the frontal lobe is associated with schizophrenia, ADHD, and dementia.

A nurse is providing a class on osteoporosis at the local seniors center. Which of the following statements related to osteoporosis is most accurate? A) Osteoporosis is categorized as a disease of the elderly. B) A nonmodifiable risk factor for osteoporosis is a persons level of activity. C) Secondary osteoporosis occurs in women after menopause. D) Slow discontinuation of corticosteroid therapy can halt the progression of the osteoporosis.

D

A nurse is reviewing the care of a patient who has a long history of lower back pain that has not responded to conservative treatment measures. The nurse should anticipate the administration of what drug? A) Calcitonin B) Prednisone C) Aspirin D) Cyclobenzaprine

D

A patient is undergoing diagnostic testing for osteomalacia. Which of the following laboratory results is most suggestive of this diagnosis? A) High chloride, calcium, and magnesium B) High parathyroid and calcitonin levels C) Low serum calcium and magnesium levels D) Low serum calcium and low phosphorus level

D

A patient presents at the clinic with complaints of morning numbness, cramping, and stiffness in his fourth and fifth fingers. What disease process should the nurse suspect? A) Tendonitis B) A ganglion C) Carpal tunnel syndrome D) Dupuytrens disease

D

Acetylcholine (ACh)

Mechanism of action: Excitory/Inhibitory Physiological effects: Controls sleep and wakefulness cycle; signals muscles to become alert

Serotonin

Mechanism of action: Inhibitory Physiological effects: Controls food intake, sleep and wakefulness, temperature regulation, pain control, sexual behaviour, regulation of emotion

Gamma-aminobutyric acid (GABA)

Mechanism of action: Inhibitory Physiological effects: Modulates other neurotransmitters

The nurse is preparing to provide care for a patient diagnosed with myasthenia gravis. The nurse should know that the signs and symptoms of the disease are the result of what? A) Genetic dysfunction B) Upper and lower motor neuron lesions C) Decreased conduction of impulses in an upper motor neuron lesion D) A lower motor neuron lesion

D) A lower motor neuron lesion

18. You are the nurse caring for an elderly patient who is being treated for community-acquired pneumonia. Since the time of admission, the patient has been disoriented and agitated to varying degrees. Appropriate referrals were made and the patient was subsequently diagnosed with dementia. What nursing diagnosis should the nurse prioritize when planning this patients care? A) Social isolation related to dementia B) Hopelessness related to dementia C) Risk for infection related to dementia D) Acute confusion related to dementia

D) Acute confusion related to dementia

Nurses and members of other health disciplines at a states public health division are planning programs for the next 5 years. The group has made the decision to focus on diseases that are experiencing the sharpest increases in their contributions to the overall death rate in the state. This team should plan health promotion and disease prevention activities to address what health problem? A) Stroke B) Cancer C) Respiratory infections D) Alzheimers disease

D) Alzheimers disease

29. You are caring for an 82-year-old man who was recently admitted to the geriatric medical unit in which you work. Since admission, he has spoken frequently of becoming a burden to his children and staying afloat financially. When planning this patients care, you should recognize his heightened risk of what nursing diagnosis? A) Disturbed thought processes B) Impaired social interaction C) Decisional conflict D) Anxiety

D) Anxiety

The nurse caring for a patient in ICU diagnosed with Guillain-Barré syndrome should prioritize monitoring for what potential complication? A) Impaired skin integrity B) Cognitive deficits C) Hemorrhage D) Autonomic dysfunction

D) Autonomic dysfunction

18. What should the nurse suspect when hourly assessment of urine output on a patient postcraniotomy exhibits a urine output from a catheter of 1,500 mL for two consecutive hours? A) Cushing syndrome B) Syndrome of inappropriate antidiuretic hormone (SIADH) C) Adrenal crisis D) Diabetes insipidus

D) Diabetes insipidus

33. A gerontologic nurse is basing the therapeutic programs at a long-term care facility on Millers Functional Consequences Theory. To actualize this theory of aging, the nurse should prioritize what task? A) Attempting to control age-related physiological changes B) Lowering expectations for recovery from acute and chronic illnesses C) Helping older adults accept the inevitability of death D) Differentiating between age-related changes and modifiable risk factors

D) Differentiating between age-related changes and modifiable risk factors

A middle-aged woman has sought care from her primary care provider and undergone diagnostic testing that has resulted in a diagnosis of MS. What sign or symptom is most likely to have prompted the woman to seek care? A) Cognitive declines B) Personality changes C) Contractures D) Difficulty in coordination

D) Difficulty in coordination

A patient with suspected Creutzfeldt-Jakob disease (CJD) is being admitted to the unit. The nurse would expect what diagnostic test to be ordered for this patient? A) Cerebral angiography B) ABG analysis C) CT D) EEG

D) EEG

37. A gerontologic nurse is overseeing the care that is provided in a large, long-term care facility. The nurse is educating staff about the significant threat posed by influenza in older, frail adults. What action should the nurse prioritize to reduce the incidence and prevalence of influenza in the facility? A) Teach staff how to administer prophylactic antiviral medications effectively. B) Ensure that residents receive a high-calorie, high-protein diet during the winter. C) Make arrangements for residents to limit social interaction during winter months. D) Ensure that residents receive influenza vaccinations in the fall of each year.

D) Ensure that residents receive influenza vaccinations in the fall of each year.

An elderly patient has come in to the clinic for her twice-yearly physical. The patient tells the nurse that she is generally enjoying good health, but that she has been having occasional episodes of constipation over the past 6 months. What intervention should the nurse first suggest? A) Reduce the amount of stress she currently experiences. B) Increase carbohydrate intake and reduce protein intake. C) Take herbal laxatives, such as senna, each night at bedtime. D) Increase daily intake of water.

D) Increase daily intake of water.

The nurse is creating a plan of care for a patient who has a recent diagnosis of MS. Which of the following should the nurse include in the patient's care plan? A) Encourage patient to void every hour. B) Order a low-residue diet. C) Provide total assistance with all ADLs. D) Instruct the patient on daily muscle stretching.

D) Instruct the patient on daily muscle stretching.

19. During the examination of an unconscious patient, the nurse observes that the patients pupils are fixed and dilated. What is the most plausible clinical significance of the nurses finding? A) It suggests onset of metabolic problems. B) It indicates paralysis on the right side of the body. C) It indicates paralysis of cranial nerve X. D) It indicates an injury at the midbrain level.

D) It indicates an injury at the midbrain level.

14. A nurse is planning discharge teaching for an 80-year-old patient with mild short-term memory loss. The discharge teaching will include how to perform basic wound care for the venous ulcer on his lower leg. When planning the necessary health education for this patient, what should the nurse plan to do? A) Set long-term goals with the patient. B) Provide a list of useful Web sites to supplement learning. C) Keep visual cues to a minimum to enhance the patients focus. D) Keep teaching periods short.

D) Keep teaching periods short.

5. A patient with a documented history of seizure disorder experiences a generalized seizure. What nursing action is most appropriate? A) Restrain the patient to prevent injury. B) Open the patients jaws to insert an oral airway. C) Place patient in high Fowlers position. D) Loosen the patients restrictive clothing.

D) Loosen the patients restrictive clothing.

27. A patient has a poor prognosis after being involved in a motor vehicle accident resulting in a head injury. As the patients ICP increases and condition worsens, the nurse knows to assess for indications of approaching death. These indications include which of the following? A) Hemiplegia B) Dry mucous membranes C) Signs of internal bleeding D) Loss of brain stem reflexes

D) Loss of brain stem reflexes

A 73-year-old man comes to the clinic complaining of weakness and loss of sensation in his feet and legs. Assessment of the patient shows decreased reflexes bilaterally. Why would it be a challenge to diagnose a peripheral neuropathy in this patient? A) Older adults are often vague historians. B) The elderly have fewer peripheral nerves than younger adults. C) Many older adults are hesitant to admit that their body is changing. D) Many symptoms can be the result of normal aging process.

D) Many symptoms can be the result of normal aging process.

The nurse is caring for a patient with increased intracranial pressure (ICP). The patient has a nursing diagnosis of ineffective cerebral tissue perfusion. What would be an expected outcome that the nurse would document for this diagnosis? A) Copes with sensory deprivation. B) Registers normal body temperature. C) Pays attention to grooming. D) Obeys commands with appropriate motor responses.

D) Obeys commands with appropriate motor responses.

A patient presents at the clinic complaining of pain and weakness in her hands. On assessment, the nurse notes diminished reflexes in the upper extremities bilaterally and bilateral loss of sensation. The nurse knows that these findings are indicative of what? A) Guillain-Barré syndrome B) Myasthenia gravis C) Trigeminal neuralgia D) Peripheral nerve disorder

D) Peripheral nerve disorder

36. The nurse is caring for a patient who has undergone supratentorial removal of a pituitary mass. What medication would the nurse expect to administer prophylactically to prevent seizures in this patient? A) Prednisone B) Dexamethasone C) Cafergot D) Phenytoin

D) Phenytoin

22. The nurse is providing care for a patient who is withdrawing from heavy alcohol use. The nurse and other members of the care team are present at the bedside when the patient has a seizure. In preparation for documenting this clinical event, the nurse should note which of the following? A) The ability of the patient to follow instructions during the seizure. B) The success or failure of the care team to physically restrain the patient. C) The patients ability to explain his seizure during the postictal period. D) The patients activities immediately prior to the seizure.

D) The patients activities immediately prior to the seizure.

17. The home health nurse is making an initial home visit to a 76-year-old widower. The patient takes multiple medications for the treatment of varied chronic health problems. The patient states that he has also begun taking some herbal remedies. What should the nurse be sure to include in the patients teaching? A) Herbal remedies are consistent with holistic health care. B) Herbal remedies are often cheaper than prescribed medication. C) It is safest to avoid the use of herbal remedies. D) There is a need to inform his physician and pharmacist about the herbal remedies.

D) There is a need to inform his physician and pharmacist about the herbal remedies.

The nurse caring for a patient diagnosed with Guillain-Barré syndrome is planning care with regard to the clinical manifestations associated this syndrome. The nurse's communication with the patient should reflect the possibility of what sign or symptom of the disease? A) Intermittent hearing loss B) Tinnitus C) Tongue enlargement D) Vocal paralysis

D) Vocal paralysis

A patient diagnosed with Bell's palsy is having decreased sensitivity to touch of the involved nerve. What should the nurse recommend to prevent atrophy of the muscles? A) Blowing up balloons B) Deliberately frowning C) Smiling repeatedly D) Whistling

D) Whistling

A nurse is providing an educational class to a group of older adults at a community senior center. In an effort to prevent osteoporosis, the nurse should encourage participants to ensure that they consume the recommended adequate intake of what nutrients? Select all that apply. A) Vitamin B12 B) Potassium C) Calcitonin D) Calcium E) Vitamin D

D, E

Histamine

Mechanism of action: Neuromodulator Physiological effects: Controls alertness, gastric secretions, cardiac simulation, peripheral allergic response

Phenytoin

Dilantin Anticonvulsant Seizures

Pseudoparkinsonism

Drug induced parkinsonism. Symptoms include stiff, stooped posture; mask-like facies; decreased arm swing; a shuffling, festinating gait(small steps); cogwheel rigidity; drooling; tremor; bradycardia; and coarse pill-rolling movements of the thumb and fingers while at rest

Client teaching for antianxiety drugs

Drugs relief symptoms but do not treat the underlining cause. They have a potent effect on alcohol (1 drink = 3 drinks). Withdrawal can be fatal. Clients should be aware of decreased response time and practice precaution.

Stimulants

Drugs that excite neural activity and speed up body functions. Primarily used for ADHD, residual attention deficit disorder in adults, and narcolepsy. Most common side effects is anorexia, weight loss, nausea, and irritability. Less common side effects are dizziness, dry mouth, blurred vision, and palpitation. Long term problems are growth and weight suppression, which can be prevented by taking "drug holidays" when not needed by the child.

Antianxiety (anxiolytic) drugs

Drugs that increase the effectiveness of gamma aminobutyric acid (GABA), a neurotransmitter that modulates excitability and anxiety. Benzodiazepines tend to cause physical dependence, these symptoms often are the same as anxiety so clients fear that they cannot handle their anxiety without medication or that they fear the return of their anxiety which can lead to overuse/abuse. Side effects most commonly associated is CNS depression, especially in the elderly which increases their risk for falls. Clients can develop a tolerance to the symptoms and they generally decrease in intensity.

Stimulant drugs

Drugs that stimulate or excite the CNS. Primary use is ADHD in children and adolescents, residual ADD in adults, and Narcolepsy

Which is the most effective way in which the nurse can assess the progress of a client's mental status based on the expected outcome of the therapeutic plan?

Evaluation

A nurse has been asked to complete a mental status examination of a psychiatric-mental health client. Which is a necessary component of this assessment?

Evaluation of insight and judgment The mental status examination is a central aspect of the psychiatric assessment process that assesses current cognitive and affective functioning through data collection on appearance, behavior, level of consciousness, speech, thought content and processes, cognitive ability, mood and affect, insight, and judgment. This assessment relies almost exclusively on observation rather than inquiry and is expected to change during treatment.

circumstantial thinking

Eventually answers question after excessive, unnecessary details

Clozapine (Clozaril)

Fewer side effects but potentially fatal side effects of agranulocytosis which is usually developed suddenly and includes fever, malaise, ulcerative sore throat, and leukopenia. Effect can occur up to 24 weeks after the initiation of therapy. Initially the patient will have WBC checked weekly for the first 6 months. the patient must have a WBC above 3500/mm^3 to obtain next week's clozapine.

Psychoimmunology

Field of study that examines the effect of psychosocial stressors on the the body's immune system

During a conversation, the client states, "It's raining outside and raining in my heart. Did you know that St. Valentine used to visit jails? I've never been to jail." The nurse can correctly identify this thought process as what?

Flight of ideas Flight of ideas is the expression of multiple, unrelated ideas in a string of statements. Neologisms are new words, circumstantiality is when the client speaks about topics that are loosely related with each other, and perseveration is the repetition of words or ideas over and over.

Neuropeptides

Mechanism of action: Neuromodulators Physiological effects: Enhance, prolong, inhibit, or limit the effect of principal neurotransmitters

A young adult client is brought to the outpatient mental health clinic by the client's father. The client was diagnosed with schizophrenia 6 months ago and has been taking medication since. The father reports the client continues to hear voices despite adhering to the medication. Which term best describes the client's abnormality of perception?

Hallucination A hallucination is a subjective sensory perception in the absence of real external stimuli. The client can hear, see, smell, taste, or feel something that does not exist in reality. In this case, the client's sister died and cannot be speaking to the client, although in the client's mind the client can hear her. This is an example of an auditory hallucination, but hallucinations can occur with any of the five senses.

Amphetamine

Has high potential for abuse, prolonged use can cause dependence.

Under which component of the psychosocial assessment should the nurse document observations concerning the client's cultural considerations?

History

A psychiatric-mental health client informs the nurse that a tornado that hit a neighboring town was the client's fault because the client dislikes a neighbor. This disturbance of thought content is known as what?

Ideas of Reference Ideas of reference include beliefs that external events have personal significance. Magical thinking includes the belief that thinking about something will make it happen. Thought broadcasting includes the belief that thoughts are known by others. Obsessions include unwanted repetitive thoughts.

A psychiatric-mental health client informs the nurse that a tornado that hit a neighboring town was the client's fault because the client dislikes a neighbor. This disturbance of thought content is known as what?

Ideas of reference Ideas of reference include beliefs that external events have personal significance. Magical thinking includes the belief that thinking about something will make it happen.

Reticular Activating System (RAS)

Influences motor activity, sleep, consciousness, and awareness.

Akathisia

Intense need to move about.

Parietal lobe

Interprets sensation, of taste, and touch. It assists in spatial orientation.

Hippocampus

Involved in memory and emotional arousal.

Hypothalamus

Involved in temperature regulation, appetite control, endocrine function, sexual drive, and impulsive behaviour associated with anger, rage, or excitement.

What is the most significant benefit of using Beck's Depression Inventory to the practice of evidence-based nursing practice?

It is a standardized, reliable depression tool. Evidence-based practice promotes the use of standardized, valid, and reliable tools, guidelines, and protocols in mental health, based on aggregate data. Tools based on the majority of the population presume similarity and stability over time.

A psychiatric-mental health nurse is feeling highly anxious before conducting an interview with a client. The nurse's experience of anxiety will impact the client assessment in which way?

It will be detrimental to the interaction by decreasing the nurse's focus and attention.

Pineal glans

Located between the two hemispheres. Is an endocrine gland that influences the pituitary gland, islets of Langerhans, para-thyroid, adrenal, and gonads.

A client with psychosis who was recently admitted to a psychiatric unit says to the nurse, "The car is red. Are you ready for lunch? My head is hurting. Dogs bark loud." The client is exhibiting which type of speech?

Loose association Looseness of association is a disturbance of thinking shown by speech in which ideas shift from one unrelated or minimally unrelated subject to another. Echolalia is the parrot-like repetition of overheard words or phrases. Clang association is a type of thinking in which the sound of a word (rhyming) substitutes for logic during communication. Neologism describes the use of a new word or combination of several words coined or self-invented by a person and not readily understood by others.

As the nurse is conducting an interview with a client with a diagnosis of schizophrenia, the client states, "Bunnies are cute as a button, buttons are on my shirt, shirts can be bought in a store." Which is a term used to describe this thought process?

Loose associations

Black box warning

Medication package inserts must have a highlighted box, separate from the text, that contains a warning about the life-threathening or otherwise serious side effect(s) of the medication

As the nurse is conducting an interview with a client with a diagnosis of schizophrenia, the client states, "Bunnies are cute as a button, buttons are on my shirt, shirts can be bought in a store." Which is a term used to describe this thought process?

Loose associations In some cases a client presents several thoughts that don't make sense in conjunction with one another. This is often seen in clients with acute exacerbations of schizophrenia and is described as loose association.

MAOIs (monoamine oxidase inhibitors)

Low incidence of sedation and anticholinergic side effects. However they are used extremely cautiously because they can cause: 1. HTN crisis if patient eats food with tyramine 2. Cannot be combined with other MAOIs, tricyclics, meperidine, CNS depressants, general anesthetics, many anti-hypertensives.

Norepinephrine (noradrenaline)

MOA: Excitatory Physiologic Effects: Causes changes in attention, learning & memory, sleep and wakefulness, mood *Most prevalent neurotransmitter in the nervous system. Excess: several anxiety disorders Deficit: Memory loss, social withdrawal, depression

Dopamine

MOA: Excitatory Physiologic Effects: Controls complex movements, motivation, cognition; regulated emotional response. Indicated in schizophrenia, other psychoses, & movement disorders such as Parkinson's

Epinephrine (adrenaline)

MOA: Excitatory Physiologic Effects: Controls fight-or-flight response

Serotonin

MOA: Inhibitory Physiologic Effects: Controls food intake, sleep & wakefulness, temperature regulation, pain control, sexual behaviors, regulation of emotions *Anxiety, mood disorders, schizophrenia, delusions, hallucinations & withdrawn behavior

Antipsychotic drugs (neuroleptics)

MOA: blocks dopamine receptors USE: Schizophrenia, psychotic episodes of acute mania, psychotic depression, and drug-induced psychosis

Food containing tyramine

Matured/aged cheese Aged meats Fava beans Tofu, soy sauce, soybeans Banana peal Overripe fruit Avocado Tap beer No more than 2 cans of beer (even if non-alcoholic)/day No more than 4 ounces of wine/day Sauerkraut Marmite Yogurt, sour cream Peanuts Brewers yeast MSG (monosodium glutamate)

Norepinephrine

Mechanism of action: Excitatory Physiological effects: Cause changes in attention, learning and memory, sleep and wakefulness, mood

Dopamine

Mechanism of action: Excitatory Physiological effects: Controls complex movements, motivation, cognition; regulates emotional response

Epinephrine

Mechanism of action: Excitatory Physiological effects: Controls fight-flight

Glutamate

Mechanism of action: Excitatory Physiological effects: Results in neurotoxicity if levels are too high

opiod analgesics

Morphine, fentanyl, codeine, loperamide, methadone, meperidine, dextromethrophan, diphenoxylate, pentazocine MOA: act as agonists at opiod receptors (mu = morphine, then enkephalin and dynorphin). to modulate synaptic transmisison - open K+ channles, close Ca2+ channels- decreased synpatic transmission. Inhibit release of NE, ACh, 5-HT, and substance P Use: pain, cough depression (extromethrophan), diarrhea (loperamide, diphenoxylate- keep it on DL that I have diarrhea), maitenance programs for heroin addicts (methadone, buprenorphine + naloxone). Tox: addiction, respiratory depression, constipation, miosis. Treat with naloxone or naltrexone (opiod recetpor antagonist)

Withdrawl

New symptoms resulting from discontinuation of drug or substance

Withdrawal

New symptoms resulting from discontinuing a drug.

Lithium

Normalizes the re-uptake of serotonin, nor-epinephrine, acetylcholine, and dopamine. Reduces the release of nor-epinephrine. Toxicity levels (1.5 mEq/L) are close to therapeutic levels (1 mEq/L). Levels should be monitored Q2-3Days while therapeutic dose (900 - 3600 mg) is determined. Then it is monitored weekly until the person is stable, then it's monitored monthly.

Anticholinergic Side Effects

Often occur with use of antipsychotic drugs and includes dry mouth, orthostatic hypotension, constipation, urinary hesitance or retention, blurred near vision, dry eyes, photophobia, nasal congestion, and decreased memory

Anticholinergics side effects

Orthostatic hypotension, dry mouth, constipation, urinary hesitance/resistance, blurred near vision, dry eyes, photophobia, nasal congestion, and decreased memory. Water or hard candy may alleviate dry mouth. Exercise, stool softners, and increasing fiber intake can help with constipation.

Tardrive dyskinesia

Permanent involuntary movements of the tongue, facial and neck muscles, upper and lower extremities, and truncal musculature. Tongue thrusting and protruding, lip smacking, blinking, grimacing, and other excessive unnecessary facial movements are characteristic. Antipsychotic medications may mask the beginning symptoms. Clients who develop TD but still need to take anti-psychotics are put on atypical anti-psychotics.

Client teaching for stimulants

Potential for abuse. KEEP OUT OF REACH OF CHILDREN, 10 day supply is fatal. Take stimulants after meals to prevent anorexia and nausea. Caffeine, sugar, and chocolate free diet.

Which would not be included as a purpose of the psychosocial assessment?

Previous compliance with treatment regimen

Antidepressant drugs

Primarily used in the treatment of major depressive lines, anxiety disorders, the depressed phase of bipolar disorder, and psychotic depression Off label uses: chronic pain, migraine headaches, peripheral and diabetic neuropathies, sleep apnea, dermatologic disorders, panic disorder, and eating disorders.

The psychiatric nurse correctly identifies the client's form of communication as circumstantiality when the client does what?

Provides long, irrelevant explanations when asked why the client abuses alcohol.

Nefazodone (Serzone)

Rare: Life-threatening liver damage > liver failure

Cholinergic

Referring to cells that use acetylcholine as their synaptic transmitter.

When asking a client to "tell me how having schizophrenia has affected your life," the nurse is assessing the client's capacity for what?

Reflective insight

Thalamus

Regulates activity, sensation, and emotion.

Extra-pyramidal system

Relays information about movement and coordination from the brain to the spinal nerves.

Client teaching for antidepressants

SSRIs: Take first thing in the morning unless sedation occurs. If you miss a dose you can take it up to 8 hours late. Cyclics: Take at night in a single dose if possible. If you miss a dose you can take it up to 3 hours late, after that omit the dose. Be careful when doing activities that need you to be alert, quick reflexes, or sharp until sedative effects are determined. MAOIs: Be aware of hyperadrenergic crisis (HTN crisis) if you consume foods with tyramine. Have a list of foods to avoid and stick to diet restrictions. Do not take additional medications unless you have checked with a physician or pharmacist.

Extra-pyramidal symptoms

Serious neurological symptoms which are side effects of antipsychotic drugs. They include: Acute dystonia Pseudoparkinsonism Akathisia

The nurse is caring for a client who has recently developed psychomotor retardation. Based on this information, which behavior would the nurse expect to see in this client?

Slowness of body movements

Locus coeruleus

Small group of nor-epinephrine producing neurons that are associated with stress, anxiety, and impulsive behaviour.

Mood stabilizing drugs

Stabilizes person's mood, prevent, or minimize the highs and lows. Lithium and anticonvulsants are drugs that stabilize mood.

Client teaching for antipsychotics

Stick to prescription, do not discontinue. Tell us about the effects and we will help patient relieve the side effects with other interventions like diet or pharmacotherapy. Use sunscreen because of increase photosensitivity. Monitor sleepiness or drowsiness. Avoid driving or doing alert demanding activities until you regain ability. If you miss a dose you can take it if its only 3-4 hours late. If it's more than 4 hours late then omit that dose and take the next one as you would have in anyway.

During an assessment, which would be the most important question topic?

Suicidal Ideation The client's safety is a priority. Asking clients clearly and directly about suicidal ideation is essential.

Post delirium/sedation syndrome

Symptoms include sedation, confusion, disorientation, agitation, and cognitive impairment which can progress to ataxia, convulsions, weakness, and hypertension (which can lead to arrest).

Pseudoparkinsonism

Symptoms mimicking Parkinson's Disease. Stiff stooped posture, masklike facies, decreased arm swing, shuffling - festinating gait, cogwheel rigidity, drooling, tremors, bradycardia, and coarse pill-rolling movements of the thumb and fingers while at rest.

Rebound

Temporary return of symptoms; may be more intense than original symptoms.

Potency

The amount of the drug needed to achieve maximum therapeutic effect.

The nurse is performing an assessment of a client with psychiatric illness. The nurse documents that the client has a restricted affect. Which behavior of the client is indicative of restricted affect?

The client displays only one type of facial expression.

The nurse is performing an assessment of a client with psychiatric illness. The nurse documents that the client has a restricted affect. Which behavior of the client is indicative of restricted affect? Choose the best answer.

The client displays only one type of facial expression.

Of the following clinical information, which one would be the most important in determining whether the client would be diagnosed with a mental disorder?

The client is unable to continue school work and has been sitting on the client's bed for 3 days.

During a mental status exam, what conclusion should the nurse draw when the client is able to complete fewer than half of tasks accurately?

The client's cognitive deficit is significant

The nurse finds that the client is constantly rubbing the hands. Under which component of psychosocial assessment should the nurse document this finding?

The general assessment and motor behavior component

Synapse

The junction between the axon tip of the sending neuron and the dendrite or cell body of the receiving neuron

When conducting a psycho-social assessment, the nurse inquires about the client's social supports. In order to effectively do this, which does the nurse need to explore?

The length and quality of relationships

Efficacy

The maximum therapeutic effect that a drug can achieve.

The nurse is assessing an older adult client with lower back pain. In the course of assessment, the nurse learns that the client lost a spouse 10 weeks ago. The client laughs inappropriately and states, "My spouse just up and left me!" Which is the nurse's best response?

The nurse should recognize the incongruity between content and behavior and find ways of exploring further.

Pons

The part of the brain stem that links the medulla and the cerebrum. A primary motor pathway.

Kindling process

The snowball like effect seen when minor seizures build up into more severe seizures.

Kindling Process

The snowball-like effect seen when minor seizure activity seems to build up into more frequent and sever seizures

Half-life

The time it takes for half of the drug to be removed from the blood stream.

Half-life

The time it takes for half of the drug to be removed from the bloodstream

Psychopharmacology

The use of medications to treat mental illness that directly affect the CNS and subsequently, behavior, perceptions, thinking and emotions

Anxiolytic drugs (anti-anxiety drug)

Used to treat anxiety, anxiety disorders, insomnia, OCD, depression, PTSD, and alcohol withdrawal

Mood-stabilizing drugs

Used to treat bipolar disorder by stabilizing the clients mood, preventing or minimizing highs & lows, and treating acute manic episodes. Ex. Lithium

Antipsychotic drugs (neuroleptics)

Used to treat symptoms of psychosis and the manic phase of bipolar disorder.

Off-lable use

Using a drug for a disease that differs from the one the one involved in the original testing and FDA approval.

Anticonvulsants

Valproic acid and topiramate increase levels of inhibitory neurotransmitter GABA. Thought to have a kindling process.

psychomotor retardation

Visible generalized slowing of movements and speech.

A nurse is seeing a client for a weekly therapeutic session in an outpatient psychiatric clinic. The client discloses to the nurse that the client often has thoughts about killing a neighbor. What should be the nurse's first response?

Warn the client's neighbor and report to the authorities.

Which question would be best for the nurse to ask in order to assess recent memory?

What did you eat for breakfast today? Recent memory or short-term memory asks the client about things and events that are happening currently. Asking the client what they ate for breakfast is testing recent memory. Asking for the client's birth date tests remote memory. How an orange and an apple are different tests a client's ability for abstract reasoning. If a client can tell the nurse why they are at the clinic, this assesses the client's orientation (location).

Off-label use

When a drug proves effective for a disease that differs from the one involved in the original testing & FDA approval

Antidepressants

a class of psychotropic medications used for the treatment of depression

thought insertion

a delusional belief that others are putting ideas or thoughts into the client's head; that is, the ideas are not those of the client

thought withdrawal

a delusional belief that others are taking the client's thoughts away and the client is powerless to stop it

Positron emission tomography (PET)

a diagnostic test used to examine the function of the brain by monitoring the flow of radioactive substances that are injected into the bloodstream (uses two photons simultaneously)

Tardive dyskinesia (TD)

a late-onset, IRREVERSIBLE neurologic side effect of antipsychotic medications. Characterized by abnormal, involuntary movements such as lip smacking, tongue protrusion, chewing, blinking, grimacing, and choreiform movements of the limbs and feet

thought broadcasting

belief that one's thoughts are broadcast directly from one's head to the external world

idea of reference

clients inaccurate interpretation that general events are personally directed to him or her, such as hearing a speech on the news and believing the message had personal meaning

The nurse assesses a client with a history of bipolar disorder. The client tells the nurse that an intelligence agency has surveillance equipment set up in the client's bathroom. The nurse is observing which thought process or content?

delusional thinking A delusion is a false fixed idea not based in reality. The information that the client has told the nurse in addition to the history of bipolar disorder indicates the client is expressing delusional thinking. Circumstantial thinking refers to when a client can respond to a question asked only after giving an excessive amount of detail. Tangential thinking refers to when the client wanders off topic when being asked a question in the assessment. With this type of thinking, the client never really provides the information that was originally requested. Loose associations refers to disorganized thinking that jumps from one idea to another with little or no evident relation between thoughts.

dyskinesia

difficult movement

loose association

disorganized thinking that jumps from one idea to another with little or no evident relation between the thoughts

broad

displaying a full range of emotion

restricted

displaying only one type of expression, usually serious or somber

Stool Softeners

docusate sodium (Colace)

antidepressant drugs

drugs used to treat depression; also increasingly prescribed for anxiety. Different types work by altering the availability of various neurotransmitters

Anticonvulsants

drugs used to treat mania and depression

Which client behavior would the nurse document as being an automatism?

drumming one's fingers on the table top

Anticholinergic effects

dry mouth constipation photophobia blurred vision Tachycardia

Side effect of Sinemet

dyskinesia

Flight of ideas

excessive amount and rate of speech composed of fragmented or unrelated ideas; racing, often unconnected, thoughts

While talking with a schizophrenic client, the nurse observes that the client is looking straight ahead, maintains no eye contact, and moves facial muscles very little, even though the client is telling the nurse about a very emotional episode the client just experienced with a roommate. When describing the client's affect, the nurse documents it as what?

flat The client's affect, or facial expression, would be described as "flat." Labile affect is the abnormal fluctuation or variability of one's expressions, such as repeated, rapid, or abrupt shifts. Constricted affect relates to a reduction in one's expressive range and intensity of affective responses. Blunted affect is a severe reduction or limitation in the intensity of one's affective responses to a situation.

word salad

flow of unconnected words that convey no meaning to the listener

A 20-year-old client who has a diagnosis of schizophrenia frequently experiences delusions of persecution. At the prompting of the client's mother, the nurse attempts to determine the character and severity of these delusions on a particular day. In doing so, the nurse is conducting what type of assessment?

focused

MS is a progressive demyelination of the nerves and patients sometimes require

frequent rest periods

clang association

groupings of words, usually rhyming words, that are based on similar-sounding sounds, even though the words themselves don't have any logical reason to be grouped together

inappropriate

incongruent with mood or situation

Anticonvulsants Uses

• Used to treat epilepsy » Seizures are physical alterations as a result of abnormal electrical discharges in the brain » Chronic


Ensembles d'études connexes

Chapter 7 - LTM: Encoding, Retrieval & Consolidation

View Set

Catastrophism / Gradualism / Punctuated Equilibrium

View Set